Acute medicine SCE

Ace your homework & exams now with Quizwiz!

Question 8 of 25 A 73-year-old lady who lives alone presents to you with a paracetamol overdose. In her history she mentions that it was an accidental overdose due to toothache and denies depression or suicidal thoughts. You notice that this is her second admission in a short space of time for inadvertent overdose of medication. You suspect she is becoming forgetful and may be showing signs of early dementia. You decide to conduct a mini-mental state exam (MMSE). What score is a recognised cut off point indicating cognitive impairment? (Please select 1 option) 24 25 26 27 28

24 This is the correct answer The total maximal score on the MMSE is 30 points. A score of less than 24 points is suggestive of dementia or delirium. Using a cut off of 24 points, the MMSE had a sensitivity of 87% and a specificity of 82% in a large population-based sample. The use of higher cut off scores on the MMSE improves sensitivity but lowers specificity. Reference: Crum RM, Anthony JC, Bassett SS, Folstein MF. Population-based norms for the Mini-Mental State Examination by age and educational level. JAMA. 1993;269:2386-91

Question 12 of 25 An 86-year-old lady has been admitted with pyelonephritis. She has a past history of type 2 diabetes, hypertension, diverticulitis and atrial fibrillation. She fulfils the diagnostic criteria for sepsis, is pyrexial and has raised inflammatory markers. Blood cultures are positive for Gram negative rods. An ultrasound of her kidneys excludes any collections. The patient is commenced on ciprofloxacin whilst awaiting sensitivities, but you plan to add gentamicin to the patient's therapy as she is unwell. Her creatinine clearance is >60 mL/min. Which dose of intravenous gentamicin would be appropriate in this patient? (Please select 1 option) 3 mg/kg 5 mg/kg 7 mg/kg 80 mg Gentamicin should not be given to this patient

3 mg/kg This is the correct answer Although there is no single study looking at the exact dose of gentamicin that should be prescribed in the elderly, gentamicin pharmacokinetics show wide inter-individual variability across all age groups and impaired gentamicin clearance is associated with impaired creatinine clearance in older people. It is common practice to dose adjust gentamicin from a 5 mg/kg dose to 3 mg/kg in patients older than 65 years of age. Drug levels to avoid drug toxicity are compulsory. The 5 mg/kg dose is commonly used for sepsis, if the creatinine clearance is appropriate. The 7 mg/kg dose tends to be reserved for neutropenic sepsis protocol. An 80 mg stat may be used during catheterisation or for end stage renal failure patients. Reference: Hilmer SN, Tran K, Rubie P, et al. Gentamicin pharmacokinetics in old age and frailty. Br J Clin Pharmacol. 2011;71:224-31. Triggs E, Charles B. Pharmacokinetics and therapeutic drug monitoring of gentamicin in the elderly. Clin Pharmacokinet. 1999;37:331-41.

Question 19 of 25 An elderly Indian lady presents with a long bone fracture after a fall. Her vitamin D level is 9 nmol/L and is classed as 'deficient'. What is the recommended vitamin D supplementation in this case? (Please select 1 option) 7-8.5 mcg daily orally 10 mcg daily orally 400 mcg daily orally 800 mcg daily orally 300,000 IU intramuscularly

800 mcg daily orally This is the correct answer A higher dose of oral vitamin D is required when plasma levels are <30 nmol/L. Vitamin D levels can be classified as follows: Serum 25(OH)D <30 nmol/L is deficient Serum 25(OH)D of 30-50 nmol/L may be inadequate in some people Serum 25(OH)D >50 nmol/L is sufficient for almost the whole population. Recommended supplementation is: 7.5-8 mcg dose is advocated for children from 6 months to 5 years 10 mcg dose is indicated for pregnant women 800-1000 mcg is suggested for adults who are deficient or insufficient. Previously fixed loading of ergocalciferol was prescribed intramuscularly, however this is no longer a part of the guidelines. The following groups of people are identified as at risk of vitamin D deficiency: Pregnant and breastfeeding women - intestinal malabsorption, for example, coeliac disease, Crohn's disease, gastrectomy Infants and children under age of 5 years - liver or renal disease Older people aged 65 years and over - medications including anticonvulsants, cholestyramine, rifampicin, glucocorticoids, iv bisphosphonates People who have darker skin - housebound individuals or those with low exposure to sunlight. Reference: Department of Health. Vitamin D - advice on supplements for at risk groups. International Osteoporosis Foundation. UK's first clinical guideline on identifying and treating vitamin D deficiency.

Question 25 of 25 Core Questions A 66-year-old man presents after an episode of unilateral limb weakness lasting for 30 minutes. On examination he is oriented to time, place and person and his speech is normal. His blood pressure is 145/80 mmHg, heart rate 80 bpm in sinus rhythm. His past medical history includes type 2 diabetes, hypertension, ischaemic heart disease and he is a current smoker. Which of the following is correct regarding ABCD2 score and prognosis for a stroke in next 48 hours? (Please select 1 option) ABCD2 score 4, 10% chance ABCD2 score 5, 20% chance ABCD2 score 5, 30% chance ABCD2 score 6, 10% chance ABCD2 score 6, 50% chance

ABCD2 score 6, 10% chance This is the correct answer The ABCD2 score is an evidence-based tool used to calculate the prognosis of a stroke after a transient ischaemic attack (TIA). Calculated risk scores are available for two days, seven days and 90 days stroke risk. The score itself can be calculated as follows: A Age >60 1 point <60 0 points B Blood Pressure >140/90 mmHg 1 point <140/90 mmHg 0 points C Clinical features Unilateral weakness 2 points Isolated speech disturbance 1 point Other 0 points D Duration of Symptoms >60 minutes 2 points 10 to 59 minutes 1 point <10 minutes 0 points Diabetes Present 1 point Absent 0 points According to the validation study, 6-7 points is high risk: 2-day stroke risk: 8.1% 7-day stroke risk: 11.7% 90-day stroke risk: 17.8% Reference: Johnston SC, Rothwell PM, Nguyen-Huynh MN, et al. Validation and refinement of scores to predict very early stroke risk after transient ischaemic attack. Lancet. 2007;369:283-92.

Question 19 of 29 Core Questions A 30-year-old Caucasian gentleman presented with a one week history of swollen ankles. He had no other symptoms apart from sweating. On examination, he had a collapsing pulse, a soft early diastolic murmur, crackles bibasally. He had a fever of 39°C, a blood pressure of 140/60 mmHg. Urine dipstick showed ++ blood. Blood tests revealed a microcytic anaemia, deranged LFTs and an elevated WCC and CRP. Which of the following is associated with a higher risk of mortality from this condition? (Please select 1 option) Abnormal BMI Abnormal serum creatinine Diabetes Elevated erythrocyte sedimentation rate (ESR) Intravenous drug use

Abnormal serum creatinine This is the correct answer This patient has infective endocarditis, as evidenced by the fever, murmur of aortic regurgitation, with associated wide pulse pressure, as well as the biochemical abnormalities. Factors associated with increased mortality (six month and discharge): Abnormal serum creatinine (raised) Abnormal serum albumin (low) Visible vegetation (did NOT affect in hospital mortality, but did affect mortality six months after discharge) Severe aortic regurgitation/severe mitral regurgitation Abnormal white cell count (increased or decreased). Factors not associated with increased mortality (six month and discharge): Age Sex BMI Premorbid conditions, for example, diabetes Elevated C reactive protein/ESR Embolic phenomena Positive culture (no difference between mortality in those with culture positive and culture negative endocarditis). Reference: Wallace SM, Walton BI, Kharbanda RK, et al. Mortality from infective endocarditis: clinical predictors of outcome. Heart. 2002;88:53-60.

Question 7 of 22 Core Questions You are asked to review a 65-year-old man with a 12 year history of type 2 diabetes who has presented to the Emergency department because of deteriorating blood glucose levels. His morning sugars are in the range of 9-13 mmol/L, and do not drop pre-meal. Post-meal he appears to gain some benefit from his sulphonylurea. Current antidiabetic medication is gliclazide 160 mg BD and metformin 1 g BD. Past medical history of note includes an inferior myocardial infarction and transitional cell carcinoma of the bladder. Examination reveals a BP of 145/82 mmHg, pulse is 75 bpm and regular. His chest is clear, abdomen is soft and non-tender. His BMI is 32. He has mild bilateral ankle swelling. The most recent HbA1c on the computer is 74 mmol/mol (8.9%) from three months ago, and his creatinine is 131 μmol/L. What is the most appropriate intervention for glucose control? (Please select 1 option) Add basal insulin Add pioglitazone Stop his metformin and start basal insulin Stop his orals and start basal bolus insulin Stop his orals and start BD mixed insulin

Add basal insulin This is the correct answer This patient has inadequate glucose control, with the greatest increase in his blood glucose levels seen in the fasting values. The most acceptable intervention at this stage is therefore likely to be basal insulin, which will improve glycaemic control at the expense of only one insulin injection per day. At this stage his mealtime glucose control still benefits from gliclazide, so it should be continued, and continuing metformin at the time of insulin initiation has been shown to limit weight gain. Pioglitazone is contraindicated in patients with a previous history of bladder cancer and not advised in those with a history of ischaemic heart disease. Stopping orals will drive greater weight gain and increased risk of hypoglycaemia post insulin initiation.

Question 3 of 19 A 54-year-old man who works in a warehouse comes to the Emergency department for review. He has intolerable pain in his left shoulder (he is right handed). He says the shoulder has become progressively more painful with limitation of movement over the past two weeks, despite having taken maximal ibuprofen and paracetamol. He does not recall any form of injury at the time the pain started. There is a past history of type 2 diabetes for which he takes metformin BD. His BP is 145/85 mmHg, general examination is unremarkable. His left shoulder is severely restricted across the range of movements, passive and active, including external rotation which is currently limiting his work, his ability to dress himself and to drive his car. What is the most likely underlying diagnosis? (Please select 1 option) Acromioclavicular joint disruption Adhesive capsulitis Brachial neuritis Cervical radiculopathy Supraspinatus tendonitis

Adhesive capsulitis This is the correct answer Adhesive capsulitis, otherwise known as frozen shoulder, is commonest in the 40-65 years age group. It occurs more commonly in women and in patients with diabetes. It is associated with a global restriction in shoulder movements, and has insidious onset, without any obvious history of injury. Pain can last for up to nine months, and when it finally subsides residual stiffness exists. Recovery from stiffness can then take up to a further one to three years. Analgesia coupled with early physiotherapy is key. Some small studies also show that early use of intra-articular corticosteroids may be useful. Some advocate manipulation under anaesthetic to increase the range of movement if conservative measures are unsuccessful. Acromioclavicular joint disruption is associated with injury and patients present with loss of the normal contour of the shoulder, a prominent clavicle, and sagging of the acromion. Brachial neuritis is associated with sudden onset severe shoulder pain, followed a few days later by flaccid paralysis of the shoulder muscles. Cervical radiculopathy is associated with neck, arm pain and neurological deficit. Supraspinatus tendernitis is associated with point tenderness over the tendon insertion.

Question 18 of 29 Core Questions A 45-year-old gentleman presents with a fever and a disseminated rash. There are no other localising symptoms. There is no history of travel or unwell contacts. Initial examination reveals a respiratory rate of 35 breaths per minute, a heart rate of 120 beats per minute, a blood pressure of 100/70 mmHg and a temperature of 39°C. Despite appropriate initial resuscitation and management, he deteriorates, and is transferred to the high dependency unit (HDU). Which of the following, according the Surviving Sepsis guidelines and bundles, should be initiated and implemented within the first three hours of patient presentation? (Please select 1 option) Administration of broad spectrum antibiotics Administration of inotropes Administration of vasopressors Insertion of a central venous line Insertion of an arterial line

Administration of broad spectrum antibiotics This is the correct answer The Surviving Sepsis Campaign bundles are the core of the sepsis improvement efforts. A "bundle" is a selected set of elements of care distilled from evidence-based practice guidelines that, when implemented as a group, have an effect on outcomes beyond implementing the individual elements alone. Using bundles simplifies the complex processes of the care of patients with severe sepsis. Each hospital's sepsis protocol may be customised, but it must meet the standards created by the bundle. Enhancing reliability of these bundle elements allows teams to focus on aspects of the changes they are implementing to create a reliable system that achieves the goal of 25% reduction in mortality due to sepsis called for by the surviving sepsis campaign. The following should be completed within three hours of patient presentation: Measure lactate level Obtain blood cultures prior to administration of antibiotics Administer broad spectrum antibiotics Administer 30 ml/kg crystalloid for hypotension or lactate ≥4 mmol/L. Within six hours of patient presentation you should apply vasopressors (for hypotension that does not respond to initial fluid resuscitation) to maintain a mean arterial pressure (MAP) ≥65 mmHg. In the event of persistent arterial hypotension despite volume resuscitation (septic shock) or initial lactate ≥4 mmol/L (36 mg/dL): Measure central venous pressure (CVP) Measure central venous oxygen saturation (ScvO2). Remeasure lactate if initial lactate was elevated. Reference: Surviving Sepsis Campaign. Bundles.

Question 20 of 29 Core Questions A 31-year-old IT manager presents with a two week history of intermittent bilateral knee swelling, fever and a pink-coloured rash. On examination, he has a fever of 39.2°C, a salmon-coloured rash on his arms and legs, cervical lymphadenopathy. His knees are not currently noted to be swollen. Initial tests reveal a mildly elevated ALT at 56 IU/L, a CRP of 400, and WCC of 20 ×109/L (neutrophilia). Initial rheumatological investigations (rheumatoid factor and ANA) are negative. What is the most likely diagnosis? (Please select 1 option) Adult-onset Still's disease CMV EBV HIV Mumps

Adult-onset Still's disease This is the correct answer Adult-onset Still's disease classically presents with the triad of spiking fevers, arthralgia and salmon-coloured rash associated with the fever. The Yamaguchi criteria are now used to diagnose adult-onset Still's disease. Major criteria: Arthralgia >two weeks Fever >39°C, intermittent, for >one week Typical rash White cell count >100,000 (80% granulocytes). Minor criteria: Sore throat Lymphadenopathy/splenomegaly Elevated LFTs Rheumatoid factor, antinuclear antibody negative. Reference: Efthimiou P, Paik PK, Bielory L. Diagnosis and management of adult-onset Still's disease. Ann Rheum Dis. 2006;65:564-72.

Question 9 of 25 You are the medical consultant on call in a hospital where there is a frailty unit. One third of emergency admissions are aged over 80-years-old. Which of the following criteria will help you to determine whether or not the elderly patients being admitted are considered to be 'frail'? (Please select 1 option) Admitted with a fractured neck of femur Aged 90 years or over Has a history of Parkinson's disease No next of kin available Suffers from constipation

Aged 90 years or over This is the correct answer It is important to keep abreast of changes in patient management; this question highlights the criteria for admission to a frailty unit. The acute physician needs to be aware of all the facilities available for patients within the healthcare system. Frailty units are a new way of caring for patients more effectively within hospital. Statistics show that a large proportion of emergency medical admissions are over the age of 65 (an older person) and a considerable number of these are frail elderly patients. The aim of the frailty unit is to enable a geriatrician review and multidisciplinary contact from the point of admission, achieving a more cost effective and holistic approach to care. There are several different criteria that aim to identify the frail older adult. Two commonly used examples of these assessments include the Fried criteria and the Bournemouth criteria. The Fried criteria encompass the following five dimensions: weight loss exhaustion weakness slowness, and low levels of activity. The Bournemouth criteria: 90 years or above 65 years from a nursing or residential home or community hospital 75 years from home with two or more pre-existing conditions: acute confusion history of falls incontinence of urine and/or faeces reduced mobility dementia (AMT less than 7) care package breakdown multiple pathology. Reference: Fried LP, Tangen CM, Walston J, et al. Frailty in older adults: evidence for a phenotype. J Gerontol A Biol Sci Med Sci. 2001;56:M146-56.

Question 4 of 22 A 60-year-old man with a history of type 2 diabetes for the past 15 years comes to the Emergency department complaining of severe burning and stinging in both legs from the feet up to the mid-shin. He says this has been coming on for months and is associated with loss of sensation in his feet. He takes BD mixed insulin and metformin for control of his glucose, and has had previous laser therapy. He also has a history of previous acute closed angle glaucoma. On examination his BP is 135/80 mmHg, pulse is 80 bpm and regular. His BMI is 32. Both of his legs are numb to the mid-shin. What is the most appropriate intervention for his leg pain? (Please select 1 option) Amitriptyline Axsain cream Carbamazepine Duloxetine Pregabalin

Amitriptyline This is the correct answer NICE recommends amitriptyline in patients who are unable to take duloxetine. In this case glaucoma is listed as a rare adverse event in association with duloxetine therapy, which drives us towards amitriptyline as the correct answer. Cases of glaucoma are also reported with tricyclic antidepressants although these are rarer versus the frequency reported for duloxetine, which drives us towards amitriptyline on balance of risk. Axsain cream is not recommended by NICE guidance, although it may be of value in patients with resistant pain. Pregabalin is an alternative second line therapy in patients with resistant symptoms despite amitriptyline or duloxetine. Reference: NICE. Neuropathic pain in adults: pharmacological management in non-specialist settings (CG173).

Question 1 of 25 Which of the following antihypertensives would you chose to commence in an older person with no comorbitities starting monotherapy for hypertension? (Please select 1 option) Amlodipine Bisoprolol Doxazosin Losartan Ramipril

Amlodipine This is the correct answer As per NICE guidance, patients over the age of 55, or black patients of African or Caribbean family origin of any age should be offered a calcium channel blocker as first step in the management of high blood pressure. If this is not tolerated it may be replaced with a thiazide diuretic drug. There is good evidence to suggest that the use of either thiazides or CCBs as may reduce risk of stroke in the elderly. Reference: NICE Pathways. Management of hypertension.

Question 4 of 29 According to NICE guidelines, which of the following people need to be actively screened for tuberculosis (TB)? (Please select 1 option) An 18-year-old from India starting university who has been in the UK for one week A 24-year-old FY1 of Asian origin, born in the UK, applying to start work, who has had a BCG vaccination with a scar A 25-year-old Pakistani nurse, born in the UK, who has been working in the NHS for five years A 35-year-old Bangladeshi man, born in the UK, applying for GP registration A neonate, born to Asian parents, who have both been in the UK for over 30 years

An 18-year-old from India starting university who has been in the UK for one week This is the correct answer Interferon gamma (IFN) test for TB does not differentiate between latent and active TB. The differentiation between active disease and latent infection is based on clinical history, examination and appropriate investigation. The IFN test looks at the response of the white blood cells of the patient to Mycobacterium tuberculosis (MTB) antigens; a patient previously exposed to MTB will react. A patient previously vaccinated with BCG will not react, as the BCG vaccination is composed of a live attenuated form of Mycobacterium bovis. The Mantoux test is a purified protein derivative test that involves injecting a small amount of tuberculin subcutaneously and monitoring the response. Anyone previously exposed to TB, including BCG vaccination, may mount a response. According to the latest NICE guidelines, the following groups of people need to be screened for latent TB: Household contacts of all patients with active TB Non-household contacts of all patients with active TB (for example, school, workplace) New entrants from high risk countries, where "high risk incidence" is defined as having more than 40 cases per 100,000 per year. Immunocompromised patients (including HIV), and Health care workers who have not had BCG vaccination or are without reliable evidence to suggest effective vaccination. Reference: Centers for Disease Control and Prevention. Tuberculosis (TB). NICE. Tuberculosis (NG33).

Question 15 of 19 Core Questions A 43-year-old lady is well known to the rheumatology department for her rheumatoid arthritis. Her maintenance medication is methotrexate 20 mg weekly, folic acid 5 mg, sulfasalazine 1 g bd, and prednisolone 5 mg od. She presents on the weekend general medical on-call with a tender and swollen left knee. There is no history of travel or trauma and she has been compliant with her medication. On examination her knee is painful and she is not able to move or weight bear on it very easily. She has a low grade pyrexia of 37.4°C. Blood tests are as follows: WCC 10 ×109/L Hb 130 g/L Plts 358 ×109/L CRP 53 mg/L How should you manage this patient? (Please select 1 option) Admit and seek orthopaedic review Analgesia and book an urgent outpatient appointment Aspirate the joint, start antibiotics and admit as inpatient Double the prednisolone dose and add analgesia, book an urgent outpatient appointment Start antibiotics, add analgesia and admit as inpatient

Aspirate the joint, start antibiotics and admit as inpatient This is the correct answer A native knee joint presenting with effusion can be aspirated by the general medical physician using an aseptic technique. A prosthetic joint or any other swollen joint requires specialist input from orthopaedics or rheumatology as appropriate. The synovial aspirate should be sent for microbiology, culture and sensitivity, crystals and acid fast bacilli. Broad spectrum antibiotic cover (for example, third generation cephalosporin) is preferred.

Question 2 of 19 You are asked to review a 52-year-old woman with a history of osteoarthritis and type 2 diabetes. She presents to the Emergency department with a painful left knee, the movement of which is restricted. She tells you that she sustained a graze to the front of the knee after falling off her bike a few days earlier. On examination her temperature is 37.9°C. There is a swelling anterior to the patella, the joint is warm and there is surface erythema. Which of the following is the most appropriate next step? (Please select 1 option) Arthroscopy and washout Aspiration of the swelling Local corticosteroid administration Plain x ray of the knee Rest and NSAIDs

Aspiration of the swelling This is the correct answer This patient has pre-patellar bursitis, which may be infective or inflammatory in origin. Given she has sustained a graze to the anterior surface of the knee, and she has a history of type 2 diabetes, an infective cause cannot be excluded. Taking this into account, diagnostic aspiration for microscopy, cell count, and protein estimation is recommended. In a patient with a history of diabetes, a broad spectrum penicillin with beta-lactamase resistance such as co-amoxiclav is recommended for treatment of septic bursitis. Pre-patellar bursitis usually responds well, either to aspiration and antibiotics in the case of infective bursitis, or to aspiration and non-steroidals in the case of inflammatory disease. Arthroscopy and bursectomy is required only in the case of bursitis which is refractory to conservative management. Local corticosteroid administration is considered only once infection has been excluded. Plain x ray of the knee is not indicated unless an underlying fracture or dislocation is suspected, and rest and NSAIDs should be instituted only once infection has been ruled out.

Question 3 of 25 A reasonably fit 85-year-old man presents to you after having a fall. You decide that he is fit for outpatient investigation but are unsure of how well he is coping at home. You want to assess his functional abilities and independence in activities of daily living before discharging him. Which of the following assessment tools would help you to assess this patient? (Please select 1 option) Barthel index Bournemouth frailty criteria Geriatric depression scale Glasgow elderly activity score Six minute walk test

Barthel index This is the correct answer The Barthel index of activities of daily living is a well known scoring system used in gerontology. The formula looks at functional criteria including: bowel and bladder function grooming feeding transferring mobility dressing stairs, and bathing abilities. This is an index that the acute physician should be aware of especially when discharging elderly patients following an acute hospital admission. Input from physiotherapists and occupational therapists is often helpful when using this tool. Reference: Mahoney FI, Barthel DW Functional Evaluation: The Bathel Index Md State Med J. 1965;14:61-5.

Question 17 of 29 A 67-year-old lady presents acutely unwell with a complicated urinary tract infection. You have started treatment with gentamicin, as advised by the microbiologists. Which of the following correctly describes the mechanism of action of gentamicin? (Please select 1 option) Binds to a ribosomal subunit, inhibiting translocation Inhibits bacterial cell wall synthesis Inhibits folate synthesis Inhibits peptidoglycan synthesis Inhibits topoisomerase, inhibiting DNA synthesis

Binds to a ribosomal subunit, inhibiting translocation This is the correct answer Antibiotics can be bacteriocidal (killing bacteria directly) or bacteriostatic (preventing their division). Aminoglycosides act by binding to the 30S ribosomal subunit and inhibit translocation. Aminoglycoside antibiotics include: amikacin gentamicin, and streptomycin. Beta lactams act by inhibiting cell wall synthesis. Beta lactam antibiotics include: penicillins cephalosporins cephamycins carbapenems monobactams, and beta-lactamase inhibitors. Glycopeptides act by inhibiting peptidoglycan synthesis. They include teicoplanin and vancomycin. Quinolones inhibit the bacterial DNA gyrase or the topoisomerase IV enzyme, thereby inhibiting DNA replication and transcription. They include ciprofloxacin, levofloxacin and moxifloxacin. Sulfonamides act by inhibiting folate synthesis. Examples include trimethoprim. Reference: UpToDate. Aminoglycosides. UpToDate. Beta-lactam antibiotics: Mechanisms of action and resistance and adverse effects.

Question 20 of 25 A 75-year-old lady who is normally well and not taking any regular medication is admitted with a fall. She is a non-smoker and consumes around 7-8 units of alcohol per week. Whilst investigating her condition you notice that her bones look osteopenic on the chest x ray. A recent dual energy x ray absorptiometry (DEXA) scan had been requested by her GP and the results are available in her notes. It shows a T score of −1.0 to −2.5. What would appropriately manage her bone mineral density (BMD) test result? (Please select 1 option) Bisphosphonate therapy Calcium and vitamin D supplements Lifestyle advice Oestrogen-progestin therapy Re-test in one year

Bisphosphonate therapy This is the correct answer BMD testing (DEXA) should take place in women over 65 years of age and older and in postmenopausal women younger than 65 years of age with clinical risk factors for fracture. Treatment is recommended for patients with a score between −1 to −2.5 and those <2.5. Pharmacological therapy includes oral bisphosphonates (weekly alendronic acid) and annual intravenous preparations (zolendronic acid). Oral vitamin D supplements are also recommended in this group, with a dose of 800 IU daily. Those who cannot tolerate bisphosphonate therapy may be candidates for raloxifene, a selective oestrogen receptor modulator (SERM), as an alternative. Oestrogen-progestin therapy is no longer first line therapy in postmenopausal women due to increased risk of breast cancer.

Question 26 of 29 A 24-year-old Punjabi lady presented with a three month history of occasional lower abdominal pain, fevers, and a sinus tachycardia. On examination, she was sweaty, had a sinus tachycardia of approximately 140 bpm, her BP was 120/80 mmHg, and she had a temperature of 40°C. Full clinical examination did not reveal any abnormalities: no lymphadenopathy, organomegaly, or joint swellings and no focus of infection. Repeated blood, urine, sputum cultures were negative. Her MRSA swab was negative. CXR was NAD, US abdomen was NAD and CT CAP showed scattered abdominal lymphadenopathy. Bloods showed: WCC 20 ×109/L CRP 300 mg/L TSH 0.005 mIU/L FT4 40 pmol/L TB PCR negative During her admission, the WCC and CRP decreased without antibiotics. However, she remained pyrexial, tachycardic and her TFTs didn't improve until the appropriate treatment was commenced. What is the treatment that has lead to her improvement? (Please select 1 option) Broad spectrum antibiotics Carbimazole and propranolol No treatment Rifampicin, isoniazid, pyrazinamide, ethambutol and pyridoxine Steroids

Carbimazole and propranolol This is the correct answer This patient has presented with acute hyperthyroidism. The nationality and the abdominal lymphadenopathy may lead one to consider the possibility of abdominal tuberculosis (TB). However, her presenting WCC and CRP are far too high for someone with TB, and they resolved spontaneously. Similarly, we have no justification from this lady's initial investigations for starting broad spectrum antibiotics. If her TFTs were normal then one might consider this possibility after discussion with the infectious diseases team and consideration of other causes of fever. Thyroid storm is a rare, life threatening condition characterised by severe clinical manifestations of thyrotoxicosis. Diagnosis is based upon clinical findings and patients with severe and life threatening thyrotoxicosis typically have an exaggeration of the usual symptoms of hyperthyroidism. The classical symptoms include: tachycardia hyperpyrexia central nervous system dysfunction (agitation, delirium, psychosis, stupor, or coma), and gastrointestinal symptoms (nausea, vomiting, abdominal pain). This patient had tachycardia, hyperpyrexia and abdominal pain. Thyroid function tests show hyperthyroidism (suppressed TSH, elevated free T4 and T3) generally comparable to that in patients with uncomplicated overt hyperthyroidism. Treatment is centred around the following: a beta-blocker to control the symptoms and signs induced by increased adrenergic tone a thionamide to block new hormone synthesis an iodine solution to block the release of thyroid hormone an iodinated radio contrast agent (if available) to inhibit the peripheral conversion of T4 to T3, and glucocorticoids to reduce T4-to-T3 conversion, promote vasomotor stability, and possibly treat an associated relative adrenal insufficiency.

Question 16 of 25 A 73-year-old Indian lady who is normally well presents to you with fever, headache, confusion and left iliac fossa pain. Her past medical history includes osteoarthritis for which she takes glucosamine. She returned from a holiday in India four days ago. She travels to India every year and stays there for four months over the winter period. On examination she is pyrexial 38.9°C, BP 110/60 mmHg, HR 53/min (she is not beta blocked). She has been a little constipated recently but on the ward she has passed some loose watery stool. There is a macular non-pruritic rash over her trunk and limbs. CXR is NAD, urine is NAD and ECG shows a sinus bradycardia of 48 bpm. Based on the above history, what antimicrobial agent would you choose to treat her with empirically? (Please select 1 option) Azithromycin Ceftriaxone Ciprofloxacin Gentamicin Metronidazole

Ceftriaxone This is the correct answer This patient is likely to have typhoid. Resistance patterns have led to a shift towards the third generation cephalosporins, azithromycin and fluoroquinolones as empiric therapy for typhoid fever whilst awaiting sensitivities on culture.

Question 1 of 19 A 23-year-old woman presents to the Emergency department with dysuria and a swollen tender right knee. She is usually well, her only medication being the combined oral contraceptive pill. She has recently returned from a company conference where she had unprotected sexual intercourse with a colleague. She is pyrexial 38.2°C, her right knee is hot, swollen and tender, with a significant effusion, and flexion is limited to 30 degrees. To which of the following treatments is she most likely to respond? (Please select 1 option) Ceftriaxone Ciprofloxacin Intra-articular corticosteroid Naproxen Prednisolone

Ceftriaxone This is the correct answer Whilst we are given limited information, we are told about dysuria, monoarthritis and an episode of unprotected sexual intercourse. Taken together these raise the possibility of gonococcal septic arthritis. Joint aspiration drainage, microscopy, culture and sensitivity are therefore appropriate, coupled with screening for sexually transmitted diseases. In this scenario, ceftriaxone would be the antibiotic intervention of choice, given potentially for up to four to six weeks. Ciprofloxacin is not indicated for the treatment of gonococcal arthritis because of potential resistance. Intra-articular corticosteroid or systemic prednisolone are not indicated until septic arthritis has been excluded. Whilst non-steroidals may be indicated for pain control, it is most important to treat underlying septic arthritis first.

Question 14 of 19 A 52-year-old woman comes to the Emergency department complaining of worsening posterior neck pain radiating to her upper arms and between her shoulder blades. There is vague numbness and tingling affecting both upper limbs. The pain has worsened over the past few months and is exacerbated by movement. Examination reveals minor neurological changes including inverted supinator jerks, and complaint of diminished fine touch in the fingers of his left hand. Plain x ray of the neck reveals disc space narrowing and osteophyte formation. Which of the following is the most likely diagnosis? (Please select 1 option) Cervical osteoporosis Cervical radiculopathy Cervical spondylolisthesis Cervical spondylosis Fibromyalgia

Cervical spondylosis This is the correct answer Neck pain is a frequent cause of presentation to the emergency department, and can be difficult to diagnose. However, here there are no signs of significant nerve compression or signs of osteoarthritic changes in the neck on x ray, and relatively stable disease. Taken together these features are consistent with cervical spondylosis, chronic cervical disc degeneration and prolapse. Four features are said to drive urgent investigation: Age of onset <20 or >55 years Weakness in more than one myotome Sensory loss in more than one dermatome, and Intractable or increasing pain. None of those features are present here apart from increasing neck pain. As such, in the first instance she can be managed conservatively with a review of her analgesia. We are not given any indication of risk factors for osteoporosis, or a history of low impact fractures elsewhere. The x ray image is also not in keeping with this as an underlying diagnosis. Cervical radiculopathy is associated with significant neurological impairment, localised to one side. Spondylolisthesis is associated with vertebral displacement and more significant neurological impairment, fibromyalgia with multiple areas of point muscular tenderness.

Question 17 of 22 A 26-year-old woman is admitted to the Emergency department. She has a history of type 1 diabetes and is currently managed with an insulin pump. She is 26 weeks pregnant with her second child. Over the past 24 hours her condition has deteriorated significantly with vomiting and increased blood glucose levels despite increasing her basal insulin rate. She feels this is due to a heavy cold. On examination she is pyrexial 38.2°C, there are signs of left lower lobe consolidation on auscultation of the chest. Her abdomen is soft but generally tender. Her bloods are shown below: Na 142 mmol/L (135-145) K 6.2 mmol/L (3.5-5.5) Bicarb 19 mmol/L (24-30) Urea 10.2 mmol/L (2.5-8) Glucose 24.2 mmol/L (<11.0) Iv normal saline is commenced. What is the most appropriate intervention for glucose control? (Please select 1 option) Change her pump catheter and site and increase the basal rate Stop the pump and give a bolus of s/c long acting insulin Stop the pump and give boluses of s/c short acting insulin Stop the pump and start an IV fixed rate insulin infusion Stop the pump and start an IV sliding scale of insulin

Change her pump catheter and site and increase the basal rate This is the correct answer This patient has deteriorating blood glucose control, yet her bicarbonate is only slightly below the normal range and her urea only mildly elevated. The fact that increasing the basal rate of the pump has not achieved glycaemic control raises the possibility of an insulin catheter blockage/pump failure. Therefore, in the first instance the initial, optimal step is a site and catheter change for the pump, continuing to deliver insulin via this route. This should of course be matched with continued IV fluid rehydration. Only if the pump fails to control glucose could the delivery method then be switched to an IV fixed rate insulin infusion.

Question 6 of 29 A patient is referred to the infectious diseases clinic by a GP for investigation of a raised ALT of 90 IU/L. You send off a hepatitis screen and obtain the following results: Hepatitis B surface antigen (HBsAg) positive Hepatitis B surface antibody (anti-HBs) negative Total hepatitis B core antibodies (anti-HBc) positive Hepatitis B core antibodies Ig M (IgM anti-HBc) negative Hepatitis B e antigen (HBeAg) negative Hepatitis B e antibody (anti-HBe) positive What do these results indicate? (Please select 1 option) Acute infection Chronic infection/carrier Immune due to previous infection Immune due to previous vaccination Susceptible to hepatitis B infection

Chronic infection/carrier This is the correct answer The presence of hepatitis B e antigen (HBeAg) is an indicator of infectivity - its presence indicates high infectivity. Hepatitis B surface antigen (HBsAg) is a protein on the outside of the virus and can be detected in the serum during either acute or chronic infection. It is absent in those who have been vaccinated against hepatitis B. The body normally produces antibodies to HBsAg as part of the normal immune response to infection. Hepatitis B surface antibody (anti-HBs) is present in those who have been vaccinated. It is present in those who have mounted a successful immune response to the virus and interpreted as indicating recovery and immunity. Total hepatitis B core antibody (anti-HBc) appears at the onset of symptoms and persists for life. Its presence indicates ongoing or previous infection with hepatitis B. IgM hepatitis B core antibody (IgM anti-HBc) indicates acute infection. It persists for around six months after infection. Reference: Centers for Disease Control and Prevention (CDC). Interpretation of Hepatitis B Serologic Test Results. World Health Organization (WHO). HBV serological markers in hepatitis patients.

Question 12 of 19 A 71-year-old man with chronic renal impairment and NYHA grade 3 heart failure comes to the Emergency department for review. He has an acutely swollen, red, left great toe and is unable to mobilise. Medication includes ramipril, furosemide, spironolactone, bisoprolol, atorvastatin and aspirin. On examination his blood pressure is 115/72 mmHg, pulse is 70 bpm and regular. There are signs of mild biventricular heart failure. His creatinine is elevated at 185 μmol/L (70-120), CRP is 185 (<10), and urate is 600 μmol/L (180-420). What is the most appropriate intervention? (Please select 1 option) Allopurinol Colchicine Naproxen Prednisolone Rasburicase

Colchicine This is the correct answer This patient has acute gout and a number of other medical problems including CKD and heart failure. Acute attacks of gout are thought to be precipitated by elevated triglycerides and uric acid, which lead to IL-1 production and subsequent acute inflammation. As such, treatments targeting inflammation are initial therapies of choice. Colchicine is the best option in patients with underlying cardiac or renal impairment (although it is contraindicated when the GFR is <10). Its mode of action is not clearly defined, although it is thought to inhibit macrophage activity. Allopurinol, whilst effective in the long term for the treatment of gout should not be started during an acute attack because of the risk of worsening symptoms due to mobilisation of tissue-bound urate. Naproxen leads to salt and water retention and because of this can worsen hypertension and symptoms of heart failure. Prednisolone can also lead to fluid retention but in this case would be considered in the event that colchicine is not tolerated. Rasburicase is recombinant urate oxidase used for pre-treatment in haematological malignancy patients receiving chemotherapy to prevent urate nephropathy. Reference: Electronic Medicines Compendium (eMC). Colchicine 500 micrograms Tablets: Contraindications.

Question 22 of 25 A 75-year-old lady presents in follow up clinic with a history of left-sided hemiplegia for 12 hours, which has now fully resolved. The event occurred two weeks ago. She was managed on the ward and discharged to follow up. On examination she is in atrial fibrillation for which she has been taking aspirin. She has been counselled for warfarin previously and is not keen to take this due to potential risks and need for regular blood tests. Which of the following are you most likely to prescribe? (Please select 1 option) Commence oral rivaroxaban Commence subcutaneous low molecular weight heparin Commence warfarin Continue on aspirin 300 mg for two weeks, then 75 mg od Hold off anticoagulation due to the risk of haemorraghic transformation

Commence oral rivaroxaban This is the correct answer This lady has suffered a cerebral ischaemic embolism secondary to atrial fibrillation and is at high risk of suffering a further event. The severity of the stroke and the chance of recurrence are both important to consider. The CHADS2-VASc score is an important evidence-based tool when evaluating anticoagulation for AF post stroke. CHADS Score CHADS-VASc Score C Congestive heart failure 1 1 H Hypertension 1 1 A Age ≥75 years 1 2 D Diabetes mellitus 1 1 S Stroke (or TIA) 2 2 V Vascular disease 1 A Age 66-74 years 1 Sc Sex category (female) 1 Risk factors for haemhorragic transformation include: the extent of the infarction use of anticoagulation within 14 days of an embolic event uncontrolled hypertension, and prior use of anticoagulants, in particular vitamin K antagonists. At the present time NICE guidance on atrial fibrillation (CG180) advocate the use of warfarin. However local guidance may vary and use of newer anticoagulants is increasing. Trials such as ROCKET AF (rivaroxaban), RE-LY (dabigatran) and ARISTOTLE (apixaban) have proven that the benefits of anticoagulation outweigh the risks of bleeding and that risk of intracerebral haemorrhage in newer oral anticoagulants is comparable to that of warfarin. More recently CG180 has introduced the use of a NOAC for anticoagulation in non valvular atrial fibrillation. Reference: NICE. Atrial fibrillation (CG180).

Question 3 of 22 You are reviewing a 54-year-old man with type 2 diabetes pre-discharge after an inferior STEMI. He has been treated with metformin and sitagliptin previously, and a most recent HbA1c is 55 mmol/mol (7.2%). His glucoses were noted to be high during the admission and he was prescribed insulin on a sliding scale. Sugars have now settled. Which of the following is the most appropriate strategy with respect to managing his glucose control post-discharge? (Please select 1 option) Add basal insulin Add bolus insulin to orals Continue oral therapy only Transition to basal bolus insulin Transition to BD mixed insulin

Continue oral therapy only This is the correct answer The first DIGAMI study originally hinted that aggressive management of blood glucose with insulin therapy at the time of an MI is associated with a positive mortality benefit. However, the study was under-powered, and was followed with DIGAMI-2, which did not suggest that insulinisation has an outcome benefit. Given this patient's HbA1c is 55 mmol/mol (7.2%), it is acceptable for him to continue oral therapy. Over the longer term, other studies that have looked at aggressive targeting of blood glucose including insulin therapy also do not support its introduction in this case. Both the VADT and ACCORD studies, which examined aggressive vs. standard blood glucose lowering therapy, suggested that targeting of HbA1c <53 mmol/mol (<7%) was associated with increased risk of sudden death. In these populations with established type 2 diabetes, the more aggressive blood glucose lowering strategy was also not associated with a microvascular benefit. Reference: Action to Control Cardiovascular Risk in Diabetes Study Group, et al. Effects of intensive glucose lowering in type 2 diabetes. N Engl J Med. 2008;358:2545-59. Duckworth W, Abraira C, Moritz T, et al. Glucose control and vascular complications in veterans with type 2 diabetes. N Engl J Med. 2009;360:129-39. Malmberg K, Rydén L, Wedel H, et al. Intense metabolic control by means of insulin in patients with diabetes mellitus and acute myocardial infarction (DIGAMI 2): effects on mortality and morbidity. Eur Heart J. 2005;26:650-61.

Question 13 of 25 A 94-year-old man with a history of CVA, right-sided hemiplegia, vascular dementia and COPD is admitted to you from his nursing home. He is bed-bound, aphasic and is now septic secondary to an infected pressure ulcer. On examination there is a stage III ulcer with slough on top and necrosis at the edges. He is pyrexial at 38.3°C, blood pressure 110/50mmHg, HR 107 in AF. Intravenous teicoplanin and gentamicin have been started as the patient is penicillin allergic. Intravenous fluids are prescribed. Blood cultures and a wound swab results are pending. Which of the following measures would most improve this patient's care? (Please select 1 option) Calcium alginate dressing Change of antibiotics Debridement and washout Nutritional support Pressure mattress

Debridement and washout This is the correct answer The presence of necrosis in a pressure ulcer or systemic sepsis secondary to an ulcer are indications for surgical intervention. Necrotic tissue promotes bacterial growth and impairs wound healing. All of the above measures would be appropriate but given the degree of sepsis in this patient, debridement of the dead tissue and washout of the area would help improve tissue penetration of the antibiotics, as well as optimising the tissue to start healing

Question 10 of 29 A 30-year-old teacher presented with a fever of 39°C and arthralgia after returning from travel in India. She had spent two months in Mumbai, India, and her symptoms started six days after her return. She had not taken any malaria prophylaxis. Initial laboratory investigations showed: thrombocytopenia slightly elevated ALT leucopenia normal renal function, and malaria optimal testing negative. Which of the following is the most likely diagnosis? (Please select 1 option) Chikungunya Dengue fever Influenza Malaria Salmonella typhi

Dengue fever This is the correct answer The combination of laboratory findings, combined with the timing of onset of symptoms is characteristic of dengue. Dengue fever is transmitted when susceptible humans are bitten by an infected female Aedes mosquito. Viraemia in humans begins towards the end of a four to six day incubation period and persists until fever abates. An uninfected Aedes mosquito may acquire the virus after feeding on an infected individual during this viraemic period. Once infected, mosquitoes carry the virus for their lifespan and remain infective for humans. Aedes aegypti mosquitoes are endemic in all continents except for Europe and Antarctica. The clinical manifestations of dengue range from self-limited dengue fever to dengue haemorrhagic fever with shock syndrome, which carries a significant mortality rate. Classic dengue fever is an acute febrile illness accompanied by headache, retro-orbital pain, and marked muscle and joint pains. Fever typically lasts for five to seven days. Haemorrhagic manifestations and thrombocytopenia can also occur in patients with dengue fever. Physical examination is non-specific, but may include a macular or maculopapular rash in approximately 50% of cases. Dengue haemorrhagic fever is the most serious manifestation of dengue virus infection and can be associated with shock.

Question 5 of 22 Core Questions A 22-year-old man with congenital cardiomyopathy and WPW syndrome presents to the Emergency department with increasing shortness of breath and tachycardia. He has an ICD for recurrent episodes of VT, and takes long term amiodarone therapy. His GP noted a free T4 of 46.2 pmol/L, and has now checked IL-6 levels (which are elevated) before referring to the endocrine clinic. On examination in the Emergency department his BP is 112/72 mmHg, pulse is 110 bpm and regular. There are bilateral crackles on auscultation of the chest and there is bilateral ankle swelling. Routine bloods are unremarkable, and his ECG reveals sinus tachycardia. What is the most appropriate intervention? (Please select 1 option) Carbimazole Dexamethasone Potassium perchlorate Propylthiouracil Radioiodine

Dexamethasone This is the correct answer The elevated IL-6 levels suggest that this is amiodarone-induced thyroiditis, leading to significant thyrotoxicosis. In this situation corticosteroids are the initial intervention of choice to reduce inflammation and hence thyroid hormone release. The biggest decision is whether or not to continue the amiodarone. In many patients with severe cardiomyopathy there is little choice as a degree of control is required to prevent frequent firing of the ICD. Patients are poor surgical candidates, although the response to radioiodine in this situation is also poor. If control cannot be achieved with corticosteroids and antithyroid therapy, and surgery is planned, then potassium perchlorate can be used as a short term option to reduce T4. Carbimazole and propylthiouracil are add-in options to corticosteroids where amiodarone-induced thyroiditis is the underlying diagnosis. Corticosteroids are initial therapy of choice in this situation. Potassium perchlorate can be useful when given short term to gain control of thyroid function before surgery as it competitively inhibits iodine uptake by the thyroid. Radioiodine is usually ineffective in amiodarone-induced thyrotoxicosis because of the iodine loading already afforded by the amiodarone itself.

Question 4 of 25 Core Questions You are called to chemically restrain an unmanageable elderly patient on the ward who is presently under section 5(2) of the Mental Health Act. He has been behaving erratically and wandering. He is most likely to be confused secondary to urinary sepsis and dehydration, however treating him for his medical condition is impossible as he is paranoid and will not let anybody approach him. After discussing the case with the psychiatric consultant you both agree that rapid tranquilisation would be most appropriate. The nurse brings you a tray of medication prescribed by the psychiatry doctor: One syringe containing haloperidol 10 mg IM, and one syringe containing procyclidine 5 mg IV. What concerns you in the above prescription? (Please select 1 option) Dose of haloperidol should be 2.5 mg Dose of procyclidine should be 10 mg Procyclidine should not be administered alongside haloperidol Route of administration of haloperidol should be intravenous Route of administration of procyclidine should be intramuscular

Dose of haloperidol should be 2.5 mg This is the correct answer Rapid tranquilisation on the ward has many potential risks. Always ensure safety before administering any drug. Keep reversal ready where appropriate in case of respiratory arrest and check the cardiac arrest trolley for airway equipment. Monitor for respiratory depression post-administration. Haloperidol IM is the preferred route as intravenous may result in arrhythmias. Procyclidine is advised to be administered alongside haloperidol to reduce the risk of dystonia and other extrapyramidal side effects. Depending on previous patient use of illicit drugs or alcohol, larger doses of haloperidol or benzodiazepines may be required to be effective. Sufficient time should be given between each dose. Medications must never be mixed in the same syringe. Reference: NICE. Violence and aggression: short-term management in mental health, health and community settings (NG10)

Question 12 of 29 A 56-year-old sheep farmer, presents with a four week history of headache rigors. On examination chest auscultation and heart sounds are normal. CT brain conducted is also normal. A lumbar puncture reveals normal cerebrospinal fluid. Blood tests are as follows: Hb 123 g/L WCC 4.8 ×109/L Platelets 158 ×109/L Na 137 mmol/L K 4.5 mmol/L Creatinine 56 μmol/L Urea 6 mmol/L Glucose 6 mmol/L Q fever serology IgG antibodies >1:1024 Urine normal What is the most appropriate treatment in this case? (Please select 1 option) Amoxicillin Amphotericin Ciproxin Doxycycline Metronidazole

Doxycycline This is the correct answer Q fever is a zoonotic infection caused by the pathogen Coxiella burnetii that has both acute and chronic manifestations. You should keep a high level of suspicion for its investigation when dealing with immunocompromised patients, those with history of close contact with animals, and pregnant women. With this patient being a sheep farmer, he is at risk of becoming infected. The presenting complaint is often non-specific with constitutional symptoms such as fever, headache and weight loss. For acute Q fever the treatment of choice is two weeks of doxycycline (adults, children over 8 years). Q fever endocarditis ought to be excluded with a transoesophageal echocardiogram; if present prolonged treatment with antibiotics should be administered.

Question 27 of 29 A 21-year-old lady, normally fit and well, presents with a two day history of dysuria, fever, vomiting and severe back pain. What is the most likely organism causing her symptoms? (Please select 1 option) E. coli Enterococcus Klebsiella pneumoniae Proteus mirabilis Pseudomonas

E. coli This is the correct answer 75-95% of pyelonephritis is caused by E. coli. Risk factors for urinary tract infection and pyelonephritis include: Diabetes Pregnancy Hospital acquired infection Renal failure Urinary tract obstruction Presence of an indwelling urethral catheter, stent, nephrostomy tube or urinary diversion Recent urinary tract instrumentation Functional or anatomic abnormality of the urinary tract History of urinary tract infection in childhood Renal transplantation, and Immunosuppression. Reference: UpToDate. Acute complicated cystitis and pyelonephritis.

Elderly

Elderly

endocrine

Endocrine

Question 3 of 29 A 48-year-old Afro-Caribbean lady presented with a two day history of headache, associated with photophobia, fever and neck stiffness. There was no associated rash and no history of recent foreign travel. She had recently been in contact with her 20-year-old daughter who had a sore throat. Her only past medical history was breast cancer; she had had a wide local excision, chemotherapy and radiotherapy. On examination there was obvious photophobia, a fever of 38.8°C. There was no focal neurology. There were no abnormalities on her FBC, CRP, or U&Es. She proceeded to have a LP which showed the following results: Opening pressure 29 cmH2O WCC 378 cells/mm3 90% lymphocytes 10% polymorphs RCC 0 cells/mm3 Protein 0.795 mg/L Glucose 4 mmol/L Serum 8 mmol/L Appearance clear What is the most likely causative organism? (Please select 1 option) Enterovirus Mumps Mycobacterium tuberculosis Neisseria Pneumococcus

Enterovirus This is the correct answer This lady has viral meningitis, as evidenced by her symptoms and CSF results (high lymphocyte count, normal glucose and slightly elevated protein). Enterovirus is the most common cause of viral meningitis. The enteroviruses include: coxsackievirus echovirus and the numbered enteroviruses. Other causes of aseptic meningitis include: HSV (generally HSV-2 in immunocompetent adults) HIV, and mumps (aseptic meningitis is the most frequently occurring extra-salivary complication occurring in mumps). CSF findings in meningitis: Pyogenic (bacterial) Tuberculous Viral ('aseptic') Appearance Often turbid Fibrin web Usually clear Predominant cells Polymorphs Lymphocytes Lymphocytes Glucose <½ plasma <½ plasma >½ plasma Protein ↑ ↑↑ N/↑ In pneumococcal meningitis one would expect a polymorphic picture in the CSF. It is important to consider this, as our patient had had contact with someone with a sore throat. Likewise, with Neisseria one would expect a polymorphic picture. Mumps would be an important differential if she had had any evidence of salivary gland swelling. Tuberculosis would be an important differential if there was a polymorphic picture and the protein was higher

Question 20 of 22 Core Questions A 45-year-old male with type 1 diabetes and with a number of complex diabetic gastrointestinal complications is noted to have a PR interval of 0.18 s, a QRS duration of 0.1 s and a QT interval of 0.48 s on routine ECG. Which one of the following drugs is most likely to be responsible? (Please select 1 option) Cimetidine Co-trimoxazole Domperidone Erythromycin Octreotide

Erythromycin Correct Erythromycin has been associated with prolonged QT interval and torsades de pointes, and is used in diabetic gastropathy, although its benefits in the condition are not entirely understood. Rarely, domperidone has been reported to prolong QT. Prolonged QT is defined as greater than 0.45 s. Other agents include amitriptyline and phenothiazines yet metoclopramide and domperidone are not associated.

Question 5 of 19 Core Questions A 22-year-old man comes to the Emergency department for review. He has been diagnosed with ankylosing spondylitis some six months earlier after presenting to the rheumatology clinic with severe back pain and early morning stiffness. Despite regular naproxen which was changed from diclofenac some 12 weeks earlier, his pain is still not controlled. On examination both forward and lateral flexion of the spine are severely limited. In addition he has evidence of bilateral policeman's heel. His erythrocyte sedimentation rate (ESR) is elevated at 80. What is the most appropriate intervention to manage his ankylosing spondylitis? (Please select 1 option) Etanercept Methotrexate Prednisolone Rituximab Sulfasalazine

Etanercept This is the correct answer This patient has significant ankylosing spondylitis disease activity, despite a trial of two different NSAIDs. He is therefore at significant risk of permanent, irreversible joint disease. For this reason early intervention with a disease-modifying agent is recommended. In this situation, because TNF-alpha is a very important driver of inflammation, use of TNF-alpha antagonists such as etanercept is supported by NICE. Of course, this patient should be assessed for tuberculosis risk prior to starting therapy. Biologicals are actually recommended ahead of traditional DMARDs in ankylosing spondylitis. Methotrexate is the default disease-modifying agent for the treatment of rheumatoid arthritis, often in conjunction with low dose prednisolone. It is dosed weekly with supplementary folic acid. Rituximab, an anti-CD20 agent which reduces B lymphocyte number and activity is used for the treatment of rheumatoid arthritis which is resistant to other therapies. Sulfasalazine is mainly considered for the treatment of psoriatic arthritis. Reference: NICE. Adalimumab, etanercept and infliximab for ankylosing spondylitis (TA143).

Question 11 of 19 A 42-year-old woman was involved in a high speed road traffic accident and sustained closed fractures to both femurs and her pelvis. She was admitted as an emergency under the trauma team for surgical reduction and fixation, however there was a delay of 36 hours prior to surgical intervention. Two days following surgery a medical opinion is sought for shortness of breath and rash. On examination the patient is tachycardic at 120 bpm, hypoxaemic with a PO2 of 9 kPa on 35% oxygen and pyrexial at 38.5°C. There is evidence of a macular rash which is petechial in nature and has been worsening since the operation. Blood tests are as follows: Hb 90 g/L WCC 12 ×109/L Plts 98 ×109/L ESR 86 mm/hr CRP 95 mg/L Na 135 mmol/L K 4.5 mmol/L Creatinine 135 μmol/L Which of the follwoing is the unifying diagnosis? (Please select 1 option) Allergic reaction to medication Disseminated intravascular coagulopathy (DIC) Fat embolism syndrome Pulmonary embolus Thrombotic thrombocytopenic purpura (TTP)

Fat embolism syndrome This is the correct answer Fat embolism syndrome is a rare but serious complication of trauma to the pelvis and long bones. The diagnosis of fat embolism syndrome is predominantly clinical with both major and minor criteria. One major and four minor criteria must be present to make a diagnosis: Major criteria Minor Criteria Respiratory symptoms Tachycardia >110 bpm Petechial rash ESR elevated Cerebral signs Pyrexial >38.5°C Radiological disease Fat globules in retina Acute drop in Hb/platelets Renal dysfunction Acute jaundice The longer the delay between fracture/repair the greater the risk of developing FES. The petechial rash occurs in 20-50 % of patients and is pathognomonic of FES. Reference: Medscape. Fat Embolism.

Question 7 of 19 Core Questions A 24-year-old single laboratory technician, originally from Saudi Arabia, presents with painful and inflamed joints in her hands and feet which have progressed over a three week period. There are also intermittent sweats and fever. She had been prescribed a course of amoxicillin by her GP but stopped taking the medication due to development of a pink macular papular rash. She has no past medical history but her family has a history of rheumatoid arthritis. There is no history of recent travel, no cough or sputum, no dysuria and no diarrhoea. On examination the patient is pyrexial at 38.8°C, the joints are tight and swollen in a symmetrical distribution. The pain is so intense it is restricting the patient's mobility. The rash is salmon pink in colour and has the following appearance: Which of the following blood tests is likely to help confirm the diagnosis? (Please select 1 option) Antistreptolysin O titres (ASOT) Blood cultures Epstein-Barr virus Ferritin HIV test

Ferritin This is the correct answer Detection of ferritin values greater than 3000 mcg/L should lead to the consideration of Still's disease in the presence of fever and in the absence of evidence for bacterial or viral infections. Juvenile onset rheumatoid arthritis (otherwise known as Still's disease) is a systemic inflammatory condition which presents with a classic triad of: fever (quotidian in nature) salmon pink rash, and arthritis. The ferritin is extremely high and can be expected to be above 10,000 mcg/L. This returns to normal after treatment. None of the characteristic antibodies for rheumatoid arthritis (rheumatoid factor, antinuclear antibody, anti-CCP) are likely to be positive. Prognosis for the disease is favourable but complications may be severe. Treatment starts with anti-inflammatories and steroids, but interleukin receptor antagonists are particularly useful.

Question 2 of 29 A 19-year-old intravenous drug user (IVDU) presented to the Emergency department acutely unwell. He had a respiratory rate of 35/min, HR of 110 bpm, BP 90/40 mmHg and a temperature of 40°C. He was complaining of fevers, night sweats, weight loss and a cough productive of yellow/green sputum. A chest x ray showed a cavitating lesion in the right mid zone. Blood and sputum cultures showed Gram positive cocci in clumps. Which antibiotic will cover the most likely organism? (Please select 1 option) Ampicillin Ceftazidime Flucloxacillin Metronidazole Piperacillin and tazobactam (Tazocin)

Flucloxacillin This is the correct answer The most likely organism here is Staphylococcus aureus, which is a Gram positive coccus. Other organisms that can cause cavitating pneumonias include: Klebsiella - Gram negative bacillus Pseudomonas - Gram negative rod, aerobe Legionella - Gram negative Anaerobes, and Mycobacterium tuberculosis - acid-fast Gram positive bacteria (no outer cell membrane, thus do not retain the Gram stain). Flucloxacillin is good against Gram positive beta lactamase producers, such as S. aureus. Ceftazidime has limited activity against S. aureus. Tazocin provides no staphylococcal cover. Ampicillin is broad spectrum and more active against Gram negative rods. Metronidazole is better against anaerobes.

Question 15 of 25 A 72-year-old man has broken his forearm following a fall. Whilst admitted, the patient's wife requests to speak with you confidentially. She describes changes in her husband's personality over the past two years that have left her worried. He was a very social person but has become socially withdrawn of late. He used to work as a college professor teaching social sciences and was very culturally aware and always polite. She describes an incident recently where he had made offensive remarks in public. She has also noticed that his level of personal hygiene is slipping. She does not know how to tackle this and is turning to you for help. From this conversation which of the following diagnosis is most likely? (Please select 1 option) Frontal lobe tumour Fronto-temporal dementia Hypothyroidism Subclinical epilepsy Vascular dementia

Fronto-temporal dementia This is the correct answer Fronto-temporal dementia (FTD) manifests progressively as personality change which eventually progresses to global dementia. This personality change is initially subtle and can manifest as apathy with social withdrawal. In contrast, some patients develop social disinhibition and impulsivity, leading them to act out in various ways, ranging from expressions of excessive sentimentality to violent acts of aggression. Some patients alternate between passive behaviour and disinhibited outbursts. Mental rigidity can also be a feature. Patients with FTD often appear inflexible in their adherence to routines, as well as unable to adapt to new situations and see another's point of view. Other changes associated with fronto-temporal dementia include: lack of insight loss of social awareness personal hygiene may be affected patients may be incontinent change in eating patterns, and emotional blunting and loss of empathy. The primary investigations would include a CT brain to exclude a space occupying lesion and blood tests to exclude metabolic causes, but from the above description, the best fit would be for FTD.

Question 16 of 22 A 48-year-old lady presents with a lump in her neck. Biopsy confirms this is medullary thyroid cancer. Further investigations confirm a diagnosis of MEN 2A, with presence of the RET oncogene. She is advised to undergo a total thyroidectomy with central neck dissection and thyroid replacement in addition to a full body CT to screen for metastases. The patient is adopted and is not in contact with her biological parents, but would like to know how the RET gene is likely to affect her 16-year-old daughter. What will you advise her? (Please select 1 option) Her daughter should be educated and self assess for lumps Her daughter should be offered a thyroid biopsy Her daughter should be offered prophylactic thyroidectomy and replacement therapy Her daughter should be screened for the RET gene Her daughter should have thyroid ultrasound scans every three years

Her daughter should be screened for the RET gene This is the correct answer Medullary thyroid cancer (MTC) is particularly aggressive clinically and when associated with multiple endocrine neoplasia types 2A and 2B is highly penetrant. The role of RET DNA testing in MTC kindreds is an established practice and helps to predict the potential phenotype of tumour in patient relatives. Phenotypical features of the tumour may include age of onset, association with other tumours and how aggressive a tumour is in nature. Prophylactic thyroidectomy is offered to first degree relatives guided by likely age of onset from RET gene analysis. Reference: Machens A, Dralle H. Prophylactic thyroidectomy in RET carriers at risk for hereditary medullary thyroid cancer. Thyroid. 2009;19(6):551-4.

Question 1 of 29 A 22-year-old lady presents with a two day history of fever and feeling generally unwell. Prior to presentation, she was witnessed to be behaving in a "strange way", very out of character and disinhibited. She then continued to deteriorate and had a tonic clonic seizure. She is not known to suffer with seizures. There was no history of travel, animal contact or other unwell contacts. She is normally fit and well. What is the most likely causative agent? (Please select 1 option) Enterovirus Herpes simplex virus (HSV) 1 Herpes simplex virus (HSV) 2 Human immunodeficiency virus (HIV) Varicella zoster virus (VZV)

Herpes simplex virus (HSV) 1 Correct A wide variety of different viruses can infect the central nervous system (CNS). Most viruses are capable of causing either meningitis or encephalitis, although in general a given virus is more likely to cause one syndrome than the other. A subset of these viruses, including mumps, measles, varicella-zoster virus (VZV), rubella, and influenza have also been associated with postinfectious encephalitis. Primary infection is characterised by viral invasion of the CNS. Neuronal involvement can be identified on histologic examination. By contrast, in postinfectious, a virus cannot be detected or recovered on histologic examination and the neurones are spared. Perivascular inflammation and demyelination are thought to be the prominent pathophysiological manifestation in this entity. In order to identify the virus, polymerase chain raction (PCR) is used. Reference: UpToDate. Viral encephalitis in adults.

Question 8 of 29 Which of the following are part of the criteria for diagnosing sepsis according the the surviving sepsis guidelines? (Please select 1 option) Arterial hypotension, MAP <60mmHg Hyperbilirubinaemia >70 µmol/L Hypoglycaemia Increased capillary refill Lactate >3.00 mmol/L

Hyperbilirubinaemia >70 µmol/L This is the correct answer Sepsis is defined as the presence (probable or documented) of infection together with systemic manifestations of infection. Severe sepsis is defined as sepsis plus sepsis-induced organ dysfunction or tissue hypoperfusion.

Question 25 of 29 A 19-year-old Caucasian girl returns from a gap year, having travelled to Nigeria and Kenya. Despite taking all the recommended precautions and malaria prophylaxis, she has developed symptoms of fever and general malaise. She was diagnosed with P. falciparum malaria, and unfortunately deteriorated and required transfer to the intensive care unit. Which of the following is a feature of severe malaria? (Please select 1 option) Diarrhoea Haematuria Hypoglycaemia Joint swelling Visual loss

Hypoglycaemia This is the correct answer Severe malaria is potentially life threatening. It requires early recognition, treatment and management on a high dependency or intensive care unit. Malaria is the tropical disease most commonly imported into the UK, with 1500-2000 cases reported each year, and 10-20 deaths. Approximately three quarters of reported malaria cases in the UK are caused by Plasmodium falciparum, which is capable of invading a high proportion of red blood cells and rapidly leading to severe or life threatening multi-organ disease. Key features of severe malaria are: impaired consciousness and seizures renal impairment (serum creatinine >265µmol/L or urine output <0.4 ml/kg/hr) acidosis (pH <7.3) pulmonary oedema or ARDS haemoglobin <80 g/L spontaneous bleeding or disseminated intravascular coagulation shock, and haemaglobinura. Reference: Lalloo DG, Shingadia D, Pasvol G. UK malaria treatment guidelines. J Infect. 2007;54:111-121.

Question 1 of 22 Core Questions A 76-year-old woman is brought to the Emergency department by ambulance. Her neighbours called the police to break in after she had not been seen for a few days. Apparently they had not seen much of her for weeks; her daughter had been calling to do her shopping, but went on holiday approximately one week earlier. On examination her BP is 155/95 mmHg, pulse is 45 bpm and regular. Her temperature is 35.1°C. There is periorbital oedema and thinning hair. She has bibasal crackles on auscultation of the chest, and is clearly obese. Arterial blood gasses reveal a normal potassium but her sodium is low at 127 mmol/L (135-145). She is unconscious and mumbles incoherently when you try to wake her. What treatment is likely to be most effective? (Please select 1 option) IV atropine IV corticosteroids IV furosemide IV T3 NG thyroxine

IV T3 This is the correct answer The presentation here is consistent with myxoedema coma, related to chronic, gradually progressive, hypothyroidism. It is thought in severe hypothyroidism that GI absorption is compromised. As such IV T3 replacement is the initial therapy of choice, followed by conversion to either oral or NG thyroxine once the patient begins to awaken. She will also require nutritional support. IV atropine is most appropriate in the situation where there is heart block and hypotension. Its short term action in a patient with well-preserved blood pressure is not relevant here. The normal potassium and weight gain is more suggestive that the low sodium is related to SIADH and hypothyroidism rather than adrenal insufficiency. There is little evidence of heart failure apart from the bibasal crackles that are consistent with decreased cardiac output due to hypothyroidism. In this situation thyroid hormone replacement is a better option than fluid off-loading. NG thyroxine is not appropriate due to poor GI absorption in myxoedema coma.

Question 19 of 22 A 23-year-old woman with a history of type 1 diabetes comes to the Emergency department having collapsed in the local supermarket. She has been feeling increasingly unwell with nausea, vomiting and lethargy over the past few weeks and has fainted on two other occasions. She takes a basal bolus insulin regime, and despite cutting back on her insulin dose over the past week has noted increasingly low blood sugars. Her weight has decreased due to not eating. On examination her BP is 95/60 mmHg, pulse is 85 bpm and regular. She looks tanned and her BMI is 19. Investigations reveal: Na 126 mmol/L (135-145) K 4.5 mmol/L (3.5-4.5) Cr 122 μmol/L (50-90) Glucose 5.4 mmol/L (<11.5) TSH 6.4 mU/L (0.5-4.5) T4 6.0 pmol/L (8.5-15.4) Which of the following is the most important intervention? (Please select 1 option) Fluid restriction IV glucose IV hydrocortisone IV normal saline IV T3

IV hydrocortisone This is the correct answer This patient presents with adrenal insufficiency and is approaching an addisonian crisis. As such the most important intervention is corticosteroid replacement, at least initially with 200 mg IV hydrocortisone. Initial starting dose of hydrocortisone orally is 15-20 mg/day. Fludrocortisone 50-100 mcg/day is started after the first few days. Fluid restriction is instituted for SIADH, but this patient is in all likelihood volume depleted, so it is inappropriate here. Whilst she has been suffering frequent hypoglycaemia, her glucose is 5.5, so IV glucose is not essential. IV normal saline is useful for rehydration in this situation, but its role is subsidiary to cortiscosteroid replacement. There are biochemical signs of hypothyroidism, although instituting thyroid hormone replacement at this stage can exacerbate the addisonian crisis. Thyroid hormone status should be rechecked a few weeks after corticosteroid replacement is instigated and thyroid hormone replacement started only if there are still signs of hypothyroidism.

Question 6 of 22 Core Questions A patient with a 40 year history of type 1 diabetes and multiple microvascular complications comes to the Emergency department complaining of paroxysms of vomiting. He admits to intermittent symptoms of vomiting, bloating and diarrhoea over the past year. He already uses an insulin pump for glucose control, and has adjusted both his background and bolus doses to take account of reduced calorie intake. On examination his BP is 115/70 mmHg, with a postural drop of 20 mmHg on standing. His creatinine has risen to 160 μmol/L from 120 μmol/L at his last clinic visit. His glucose is 12.2 mmol/L, CRP is 12. What is the most appropriate intervention? (Please select 1 option) IV cyclizine IV erythromycin IV metoclopramide IV ondansetron IV promethazine

IV metoclopramide This is the correct answer Given the long history of diabetes and the presence of microvascular complications, it is likely this patient has gastroparesis. In this situation a prokinetic antiemetic is the best option. From the options given metoclopramide is the appropriate choice; IV erythromycin can be added if symptoms do not resolve. Chronic prescription of metoclopramide is not an option due to the risk of tardive dyskinesia. For this reason domperidone is the usual chronic therapy of choice. Unfortunately despite medication, many patients suffer symptoms over the long term. Cyclizine is an antihistamine antiemetic which is centrally acting. IV promethazine is a phenothiazine antiemetic, primarily centrally acting. As such, neither carries a high degree of efficacy in gastroparesis. Ondansetron is a centrally acting antiemetic which acts via the 5-HT system and so is not effective in gastroparesis. Eerythromycin is the add-in therapy of choice in gastroparesis.

Question 10 of 22 A 22-year-old woman who has had type 1 diabetes for 15 years presents to the Emergency department with vomiting, polyuria, polydipsia and increasing drowsiness. She has had a viral upper respiratory tract infection over the past few days. On examination her BP is 100/60 mmHg, pulse is 95 bpm and regular, her temperature is 38.1°C. Her respiratory rate is 33. Abdomen is soft but generally tender. Venous blood gas reveals: pH 7.25 (7.36-7.44) Bicarbonate 15 mmol/L (24-30) Potassium 6.2 mmol/L (3.5-5) Sodium 144 mmol/L (135-145) Glucose 43 mmol/L - Which of the following is the most appropriate next intervention? (Please select 1 option) Insulin sliding scale IV calcium gluconate IV normal saline IV sodium bicarbonate Salbutamol nebuliser

IV normal saline This is the correct answer Joint British Societies (JBS) guidelines on the management of diabetic ketoacidosis (DKA) recommend aggressive rehydration with normal saline as the initial management of choice in DKA. This is because patients presenting with DKA have a significant fluid deficit. Whilst replacing insulin is of course important, the fluid deficit in DKA can exacerbate tissue hypoperfusion and worsen both renal impairment and acidosis. Insulin sliding scale is no longer recommended by guidelines for the management of DKA. A fixed rate insulin infusion is now the treatment of choice. It is thought that a fixed rate insulin infusion leads to smoother attainment of glucose control in DKA, without swings between hypo- and hyperglycaemia. Calcium gluconate is administered in life threatening hyperkalaemia. In this situation, once insulin and rapid rehydration is commenced, potassium will fall very quickly. As such treatments to lower potassium such as calcium gluconate, or salbutamol via nebuliser are not usually required. Sodium bicarbonate can worsen tissue hypoperfusion, and paradoxically worsen acidosis. It is not therefore recommended in this situation. Reference: Savage MW, Dhatariya KK, Kilvert A, et al. Joint British Diabetes Societies guideline for the management of diabetic ketoacidosis. Diabet Med. 2011;28:508-15.

Infection

Infection

Question 28 of 29 A 29-year-old Punjabi lady presented with a five day history of a neck lump, associated with fever, malaise, headache and night sweats. Her only past medical history was of ulcerative colitis, on azathioprine. On examination there is a single, isolated palpable lymph node in the anterior cervical chain. She has a fever of 38.1°C and a slight tachycardia of 110 bpm. Initial investigations: CRP 100 mg/L ESR 50 mm/hr WCC 14.0 ×109/L Lymphocytosis Further investigations revealed: ANA negative pANCA mildly positive EBV IgG positive IgM negative Lymph node excision biopsy revealed a histology of paracortical necrosis with reactive histiocytes, CD8 and CD 4 positive lymphocytes. What is the most likely diagnosis? (Please select 1 option) Epstein-Barr virus Kikuchi's disease Lymphoma Systemic lupus erythematosus Tuberculosis

Kikuchi's disease This is the correct answer Kikuchi's disease is a rare necrotising lymphadenitis. It is a rare benign condition which normally presents with fever and cervical lymphadenopathy. The histology differentiates it from more severe conditions, which it can mimic. Differential diagnosis includes: Epstein-Barr virus (EBV) Yersinia Cytomegalovirus (CMV) Human immunodeficiency virus (HIV), and Toxoplasmosis. Some patients who were initially diagnosed with Kikuchi's disease subsequently develop systemic lupus erythamatosis (SLE), thus one should monitor the ANA and ant- dsDNA. Other symptoms which can occur in Kikuchi's include: fatigue malaise rash arthritis nausea, and hepatosplenomegaly. Investigations should include an antinuclear antigen (ANA) to differentiate the condition from SLE. Histology of lymph node excision biopsy will show paracortical foci often with necrosis and a histiocytic cellular infiltrate. The treatment is generally supportive and it is a self limiting condition. In active EBV one would expect the IgM to be positive. Lymphoma is not the case here as it is a very acute history. The histology shows reactive histiocytes, which is more suggestive of Kikuchi's. In addition, there are no Reed-Sternberg cells. Systemic lupus erythematosus is incorrect as there is only isolated lymphadenopathy. In addition, the histology does not show any neutrophils, which would be seen in SLE. It is difficult to distinguish the two, and one should monitor for the potential development of SLE. Although the history here is suggestive of TB, the histology is not. The histology in TB would show caseous necrosis. Reference: Dorfman RF, Berry GJ. Kikuchi's histiocytic necrotizing lymphadenitis: an analysis of 108 cases with emphasis on differential diagnosis. Semin Diagn Pathol. 1988;5:329-45.

Question 8 of 19 A 51-year-old man presents to the Emergency department with a hot and swollen right knee. He has recently been prescribed a course of co-amoxiclav for community acquired pneumonia some 48 hours earlier. He takes ramipril for hypertension, but nil else of note. On examination he is pyrexial 38.1°C, his BP is 128/82 mmHg, pulse is 80 and regular. His right knee is hot, swollen and erythematous, and held in flexion at 40 degrees. Bloods reveal an elevated white count and a CRP of 215. Knee x ray reveals an effusion but nil else of note. What is the most appropriate next step? (Please select 1 option) IV benzylpenicillin Knee aspiration Oral colchicine Oral naproxen Oral prednisolone

Knee aspiration This is the correct answer The implication here is that this man has pneumococcal septic arthritis related to haematogenous spread from his community acquired pneumonia. It is important however to confirm the diagnosis, and the only practical way to do this is via knee aspiration. The risk of not confirming the diagnosis is the risk of significant joint damage. It is possible that the joint fluid white cell count will be equivocal, particularly after 48 hours of antibiotic therapy. However, culture/pneumococcal PCR is useful to confirm the diagnosis. In the event of further fluid accumulation or a poor response to initial aspiration, formal surgical washout may be considered. Whilst IV benzylpenicillin is likely to be effective, you are making an assumption as to the microbiological diagnosis. Oral colchicine is used in the initial treatment of gout, and patients in the inflammatory phase of pseudogout may also see a response to therapy. Naproxen may be useful for analgesia, although it clearly would not treat the underlying cause of the septic arthritis, and there is no indication for prednisolone here.

Question 4 of 19 A 62-year-old man with poorly controlled type 2 diabetes comes to the Emergency department with a swollen deformed left ankle. He takes BD mixed insulin and metformin and has hypertension and peripheral neuropathy with numbness to the mid-shin bilaterally. He tells you the only pain in the joint is a minor dull ache. Examination reveals a warm, swollen, grossly deformed joint. He has collapse of the arches on both feet. Which of the following is the most useful investigation to differentiate between Charcot's and osteomyelitis? (Please select 1 option) Blood cultures CRP Labelled white cell scan MRI Plain x ray

Labelled white cell scan This is the correct answer Confirmation of a diagnosis of Charcot's is difficult. Often patients present late with only vague symptoms of pain but with a severely deformed joint due to underlying neuropathy. In this situation an indium labelled triple phase white cell scan can differentiate between osteomyelitis and Charcot's and can therefore dictate management (antibiotics and orthopaedic review vs. bisphosphonate and a boot style cast). Blood cultures are unlikely to prove positive in the absence of overwhelming infection and so would not be useful here. CRP is non-specific and may indicate inflammation or infection in this situation. Whilst plain x ray and MRI may demonstrate significant disruption of joint architecture, they are still not specific for differentiating between Charcot's and osteomyelitis.

Question 22 of 22 Core Questions Which of the following may be responsible for a hypokalaemic hypertension? (Please select 1 option) Bartter's syndrome Diabetic nephropathy Liddle's syndrome Non-classical congenital adrenal hyperplasia Type IV renal tubular acidosis (RTA)

Liddle's syndrome This is the correct answer Liddle's syndrome is typically associated with hypokalaemic hypertension and low renin and aldosterone concentrations - the so-called pseudo-hyperaldosteronism. Bartter's syndrome is associated with hypokalaemia, though hypertension is not a feature. In type IV RTA, there is a hyporeninaemic hypoaldosteronism, which may also be produced with diabetic nephropathy, hence hyperkalaemia is more typical.

Question 16 of 19 A 54-year-old man presents to the Emergency department with acute severe lower back pain radiating down both legs. He says this pain began when he was trying to lift a wardrobe at his daughter's new flat. When you see him in the department he is clearly in pain and lying flat on the examination couch. He has pain on straight leg raise and bilateral weakness of ankle dorsiflexion. There is diminished perianal sensation although tone appears preserved. Which of the following is the most important investigation or management step? (Please select 1 option) Bed rest Benzodiazepines Lumbar spine x ray MRI spine Opiate analgesia

MRI spine This is the correct answer The most likely diagnosis is a lumbar disc prolapse. The loss of perianal sensation raises the possibility of cauda equina syndrome, therefore evaluation via MRI is a crucial step. This is followed by urgent neurosurgical review and possible discectomy. If cauda equina syndrome is ruled out then adequate analgesia, +/- benzodiazepines to reduce muscle spasm, coupled with early physiotherapy are important steps in recovery from lumbar disc prolapse. There is no evidence that initial bed rest impacts positively on outcome in this case. A randomised controlled trial in lumbar disc prolapse without alarm symptoms showed no significant difference in outcomes between bed rest, mobilisation and alternative therapies such as osteopathy. Lumbar spine x ray is of very limited value in evaluating disc lesions, although of course it may be useful in ruling out any underlying bony lesion.

Question 11 of 29 A 73-year-old man, with a background history of hypertension, polymyalgia rheumatica and chronic back pain presents with fevers and acute on chronic back pain. There are no other localising signs or symptoms: no chest signs, no abnormal heart sounds, no splinter haemorrhages or vasculitic lesions, no abdominal signs, no joint swelling, no rash and no neurological signs. Despite various courses of antibiotics, including meropenem, ciprofloxacin, benzylpenicillin and flucloxacillin, he continues to spike fevers and have rising inflammatory markers. The following cultures have been sent: blood, urine, sputum, atypical screen. He is HIV negative, and his TFTs normal. The only positive culture is an Enterococcus faecalis from blood cultures. Which of the following investigations is most likely to yield the diagnosis in this patient? (Please select 1 option) Autoimmune screen MRI whole spine Repeated blood cultures Transoesophageal echo Transthoracic echo

MRI whole spine This is the correct answer Enterococcus faecalis is a Gram positive organism, which is commonly responsible for infective endocarditis, soft tissue infections (including discitis), and urinary tract infections. It is a common nosocomial pathogen. A transthoracic echo is not very useful in trying to identify any vegetations that may be present in infective endocarditis. If one is clinically concerned about infective endocarditis, then a transoesophageal echo (TOE) should be performed. Although E. faecalis is a common cause of infective endocarditis, this patient does not have any signs suggestive of this. This patient has discitis, caused by E. faecalis. Although this patient should have a TOE, the question asks which investigation will yield the diagnosis in this patient. An MRI whole spine is the "best fit" answer as the only telling sign in the history is that of chronic back pain. Both repeated blood cultures and an autoimmune screen are important; however they will not yield a diagnosis.

MSK

MSK

Question 19 of 19 Core Questions A 41-year-old woman is referred to you by the orthopaedic surgeons. She presented to the Emergency department with pain over the lateral epicondyle of her right elbow after playing tennis. She says the pain has significantly worsened over the past week as she has been practising for the local tournament. On examination there is pain over the lateral epicondyle to palpation, and pain is reproduced on resisted extension of the middle finger. Which of the following treatments is most appropriate? (Please select 1 option) Local corticosteroid injection Manipulation by a physiotherapist Oral NSAIDS Topical NSAIDS Use of an orthotic device

Manipulation by a physiotherapist This is the correct answer This patient has symptoms and signs consistent with tennis elbow. Over the first six weeks physiotherapy is associated with the best outcomes versus a 'wait and see' approach. Physiotherapy is also associated with better outcomes versus corticosteroids over the longer term. As such it is the preferred intervention. Whilst local corticosteroid injection impacts significantly on symptoms during the first six weeks, it is inferior over the longer term to either physiotherapy or to a non-interventional, 'wait and see' approach. Oral and topical NSAIDs are not thought to be effective over the longer term. Although some patients claim improvements in symptoms from using an orthotic device, longer term RCT data to support their use are sadly lacking.

Question 17 of 19 A 52-year-old electrician with a history of type 2 diabetes comes to the emergency department with a letter from his GP. His GP is concerned because the patient reports pain and tingling in his hands in the middle of the night, has reduced grip strength, and has been told he cannot be seen in the rheumatology clinic for three months. On examination he has weakness of thumb apposition and grip strength. Phalen's test is positive. Which of the following nerves is affected? (Please select 1 option) Radial nerve Median nerve Ulnar Nerve Axillary nerve Musculocutaneous nerve

Median nerve This is the correct answer This patient has symptoms and signs consistent with carpal tunnel syndrome, where the median nerve is compressed as it passes through the carpal tunnel. Type 2 diabetes is associated with increased risk of carpal tunnel syndrome, and his job may have contributed to increased risk of the disorder, although there may be limited scope for him to modify his pattern of work. Local corticosteroids have been shown to impact positively on symptoms for at least four weeks, although data on response beyond this time period are variable. Surgical decompression is an option if symptoms return following local corticosteroid injection.

Question 7 of 25 A patient with a history of mild Alzheimer's disease is admitted with a deterioration of his symptoms. He is currently on rivastigmine. The patient's son is the main carer and tells you that his father has become unmanageable at home, despite taking his medications regularly. You diagnose progression of his condition to "severe Alzheimer's disease" and discuss social care options with the son, as well as advise a review by a psycho-geriatrician. Which of the following drugs would you prescribe for his condition? (Please select 1 option) Donepezil Galantamine Haloperidol High dose rivastigmine Memantine

Memantine This is the correct answer Rivastigmine, donepezil and galantamine are licensed for mild-moderate Alzheimer's disease. Antipsychotics are not routinely used to treat dementia, unless there are symptoms of aggression or disruptive behaviour. With a growing cohort of elderly patients being admitted into acute care beds, it is important for an acute medicine specialist to have an overview of the drugs used in this common condition. Reference: NHS Choices. How is dementia treated?

Question 5 of 29 A 26-year-old lady presents with a two day history of sore throat, fever and difficulty swallowing. On examination, her tonsils are enlarged, swollen and purulent, there is no focal visible abscess. She is commenced on intravenous benzylpenicillin, however her symptoms have not improved over 72 hours. She remains febrile and continues to complain of a sore throat. Which of the following antibiotics should be added on? (Please select 1 option) Clarithromycin Doxycycline Flucloxacillin Gentamicin Metronidazole

Metronidazole This is the correct answer Always consider non-streptococcal causes of tonsillitis in patients who do not respond to appropriate antibiotics. Group A Streptococcus is the most common cause of bacterial sore throat. Other bacterial causative organisms include: Non-group A Streptococcus (group C or G) Arcanobacterium haemolyticum Corynebacterium diphtheriae Neisseria gonorrhoeae, and Fusobacterium necrophorum. In patients who do not respond to penicillin-based antibiotics, always consider the above organisms, and the possibility of a peritonsillar abscess. Lemierre's disease is a rare condition, most often caused by Fusobacterium necrophorum. The basic pathophysiology of this condition includes development of a peritonsillar abscess, with seeding of bacteria to the jugular vein and subsequent sepsis. Reference: Alherabi A. A case of Lemierre syndrome. Ann Saudi Med. 2009;29:58-60. UpToDate. Evaluation of acute pharyngitis in adults.

Question 14 of 29 Core Questions A 17-year-old girl presents with a vaginal discharge. It is foul smelling and described as "fishy". There is no associated itch. On examination, there is no inflammation of the vagina or cervix, and no obvious visible discharge. What is the most appropriate treatment? (Please select 1 option) Azithromycin Ciprofloxacin Clotrimazole Doxycycline Metronidazole

Metronidazole This is the correct answer It is important always to take a thorough sexual history, not only in those presenting with genital discharge, but in anyone who has any signs and symptoms of infection. Bacterial vaginosis is a sexually transmitted cause of vaginal discharge. Causative organisms include: Gardnerella vaginalis Mycoplasma hominis and anaerobes. It is caused by a change in the bacterial flora of the vagina, with a reduction in lactobacilli (important to maintain the acidic pH of the vagina), and an increase in anaerobes. Untreated, infection resolves spontaneously in a third of women. Treatment can include metronidazole or clindamycin.

Question 23 of 29 An 86-year-old gentleman presented with a fever of 38°C. There were no other focal symptoms. He had an extensive past medical history including pulmonary hypertension with mitral regurgitation (MR) and tricuspid regurgitation (TR), aortic stenosis, and previous total knee replacement (TKR). He had been under the care of the haematologists, cardiologists and neurologists who were investigating his chronic microcytic anaemia, shortness of breath and leg weakness. They had found no unifying diagnosis in three years. Clinically, there was no focus of infection on full cardiovascular, respiratory, gut, joint and neurological examination. He continued to spike fevers. Blood cultures grew Staphylococcus aureus from both bottles. Extensive investigation, including TOE, knee x ray, CXR, full septic screen, myeloma screen, autoimmune screen did not reveal the source. A review of previous investigations from four years ago revealed: CT CAP 2.9 cm saccular aortic aneurysm MRI knee No evidence of osteomyelitis TOE AS and MR, TR and pulmonary hypertension Repeat imaging confirmed the diagnosis. From this information, which of the following is the most likely diagnosis? (Please select 1 option) Infective endocarditis Malignancy Mycotic aneurysm Occult abscess Osteomyelitis of the TKR

Mycotic aneurysm This is the correct answer This gentleman had a repeat CT chest/abdo/pelvis which showed a 7 cm infected saccular aneurysm, which was responsible for his Staphylococcus bacteraemia and fevers. He had a TOE which was negative and knee x ray which did not show any osteomyelitis. The spread of this gentleman's infection was following his TKR, during which the Staph aureus was acquired, and then led to a chronically infected saccular aneurysm, responsible for his microcytic anaemia. Other sources of infection of aneurysms include direct bacterial inoculation and seeding during surgery. All the options here are valid answers. However, mycotic aneurysm is the most appropriate as with osteomyelitis one would expect to see some joint pain, and some changes on the knee x ray. With infective endocarditis one would perhaps expect some other signs of cardiovascular failure or some vasculitic phenomenon. Occult abscess would tend to have some focal symptoms such as abdominal pain. Common areas for occult abscess include psoas muscle and subphrenic.

Question 18 of 19 A 34-year-old man comes to the Emergency department for review. He suffered a bout of Campylobacter enteritis some two weeks ago and now complains of multiple joint pains, red and painful eyes, and pain on passing urine. He has a history of inflammatory bowel disease but has not suffered an exacerbation in ten years and takes a small dose of mesalazine each day. Examination reveals a symmetrical polyarthritis predominantly affecting the lower limbs. He has hyperkeratotic skin over his feet and pain over both heels on palpation. What is the most appropriate next intervention? (Please select 1 option) Ciprofloxacin Etanercept Increased mesalazine Naproxen Prednisolone

Naproxen This is the correct answer This man has reactive arthritis (formerly known as Reiter's syndrome) which is characterised by the triad of conjunctivitis, urethritis and polyarthritis. The skin changes are consistent with keratoderma blenorrhagica. The infection has long since cleared, therefore initial treatment of choice is a NSAID such as naproxen. Low dose prednisolone may be added in the event of a poor response, and in this case the mesalazine may also be increased. There is no indication of persistent infection here, therefore there is no value in a course of ciprofloxacin. Anti-TNF agents such as etanercept are usually given for psoriatic arthritis or for ankylosing spondylitis which does not respond to NSAIDs. Second line agents such as 5-ASA compounds and low dose prednisolone are used in the event of a suboptimal response to the NSAID.

Question 5 of 25 A normally active 78-year-old man presents to you with the history of an isolated seizure, unwitnessed but with loss of consciousness. His past medical history includes hypertension for which he is on ramipril. An ECG reveals a left bundle branch block which is long standing. Which of the following applies with regard to his driving licence? (Please select 1 option) Need to notify DVLA and stop driving for six months Need to notify DVLA and stop driving for 12 months No need to notify DVLA as isolated seizure No need to notify DVLA until a diagnosis is confirmed Only need to notify DVLA if the episode reoccurs

Need to notify DVLA and stop driving for six months This is the correct answer Guidance from the Driver and Vehicle Licensing Agency (DVLA) states: "A person who has suffered from a single unprovoked epileptic seizure (isolated fit) will qualify for a driving licence if he or she has been free from further attacks for a six month period, provided there are no further clinical factors or investigations that may suggest an unacceptably high risk of a further seizure occurring in which case it shall be 12 months of driving."

Question 9 of 29 A 25-year-old Punjabi lady presented to the Emergency department with her first tonic-clonic seizure. After a post-ictal period, full neurological examination did not reveal any focal abnormality. Her baseline observations were all within the normal parameters. Further questioning revealed she had been in the UK for 12 months, had no previous TB contacts, no illicit drug use and was vegetarian. Initial investigations: HIV negative, FBC- NAD, urine dip- NAD. Lumbar puncture revealed: WCC 5 cells/mm3 RCC 1220 cells/mm3 Protein IgG 0.3 mg/L Glucose 4 mmol/L Serum 7 mmol/L An MRI head confirmed a small isolated calcified ring enhancing lesion in the left parietal lobe in the brain parenchyma. There was no surrounding oedema and no evidence of raised ICP. What is the most likely diagnosis? (Please select 1 option) Cryptococcosis Neurocysticercosis Primary tumour Toxoplasmosis Tuberculosis

Neurocysticercosis This is the correct answer Neurocysticercosis is caused by the pork tapeworm, Taenia solium. The infection is spread by ingestion of the eggs of T. solium and shed in the stool of a human tapeworm carrier. Because of this it is not entirely relevant that our patient is vegetarian. Seizures are the most common initial presentation. In the nodular-granular phase of the condition, the cysts are ring enhancing and there is minimal surrounding oedema. The cysts will ultimately either resolve or calcify; it is the calcification that is associated with recurrent seizures. The symptoms and signs observed in neurocysticercosis depend upon the location of the cysts, with the parenchymal cysts being associated with seizures and headaches, and the extraparenchymal cysts being associated with symptoms of elevated intracranial pressure. Diagnosis is largely based on the clinical presentation and radiological findings (CT scanning is usually sufficient). Serology can also be sent. Treatment includes anti-convulsants, steroids and albendazole. In this case the patient is not immunocompromised. If she were HIV positive, or if there was any other indication of immune compromise, cryptococcosis and toxoplasmosis would be much higher on the differentials list. The patient has a relatively normal CSF examination. In TB, one would expect a higher protein count and a lymphocytosis.

Question 2 of 22 A 73-year-old retired accountant presents to you with mild headaches and disorientation which has been getting worse over a few days. She is becoming increasingly drowsy and you are very concerned. She has just returned from a holiday overseas. Significant past medical history includes congestive cardiac failure, chronic hyponatraemia (SIADH), osteoarthritis and gout. Medications include furosemide, tolvaptan, paracetamol and allopurinol. She is teetotal and a non-smoker. Observations reveal that she is apyrexial, with a HR of 95/min, BP 112/60 mmHg, SpO2 95% air, RR 23, BM 6 and GCS 13/15. Blood results show: Hb 120 g/L WCC 7 ×109/L Plts 220 ×109/L Na 139 mmol/L K 4.8 mmol/L Creat 275 µmol/L Urea 19 mmol/L Liver function is normal. She is sent for an urgent CT head scan. What is the scan most likely to show in this case? (Please select 1 option) Osmotic demyelination syndrome Meningitis Pituitary apoplexy Subarachnoid haemorrhage Venous sinus thrombosis

Osmotic demyelination syndrome This is the correct answer A well-documented side effect of tolvaptan is an erratic rise in serum sodium which may correct hyponatraemia too quickly resulting in cerebral demyelination. This is particularly hazardous in dehydrated patients in addition to alcohol dependent and malnourished patients. Tolvaptan is a selective, competitive vasopressin receptor 2 antagonist used to treat hyponatraemia associated with congestive heart failure, cirrhosis (cautious use in liver impairment) and the syndrome of inappropriate antidiuretic hormone. Use is limited in the UK due to cost implications. Though a highly effective treatment for patients with hyponatraemia, it may correct the sodium too quickly resulting in lethal cerebral oedema. In this patient her recent trip to the hot climate in the Bahamas, along with a loop diuretic and allopurinol may have been sufficient to induce acute kidney injury, which further contributed to unsafe correction of her hyponatraemia. Meningitis is unlikely due to apyrexia, no mention of rash or neck stiffness and normal white cell count. Sub- arachnoid haemorrhage and venous sinus thrombosis are both possible but are more likely to present with a history severe headache of sudden onset. Likewise pituitary apoplexy is likely to present with sudden onset and marked visual disturbances. Reference: Electronic Medicines Compendium (eMC). Summary of Product Characteristics: Samsca 15 mg and 30 mg tablets.

Question 24 of 29 A 64-year-old lady presents with profuse watery diarrhoea, severe abdominal pain and fever. There is no blood in the stool. There is no history of recent travel, unwell contacts or any consumption of takeaway food. She has a past medical history of osteoarthritis and recurrent urinary tract infections for which she is on low dose prophylactic antibiotics. On examination, she looks unwell, she has a diffusely tender abdomen. Per rectal examination does not reveal any bleeding. Her BP is 110/70 mmHg. Initial investigations reveal a white cell count of 30 ×109/L, CRP of 200 mg/L and a urea of 15 mmol/L and creatinine of 300 µmol/L. Abdominal radiograph is normal. Which antibiotic is most likely to be effective? (Please select 1 option) PO metronidazole 400 mg TDS for 10-14 days PO vancomycin 125 mg QDS for 10-14 days PO vancomycin 500 mg QDS for 10-14 days IV metronidazole 500 mg TDS for 10-14 days IV vancomycin 500 mg QDS for 10-14 days

PO vancomycin 125 mg QDS for 10-14 days This is the correct answer This lady has features of severe Clostridium difficile infection (CDI). Clostridium difficile is a Gram positive spore-forming bacterium, and produces signs and symptoms via toxin formation. The most common symptoms are profuse watery diarrhoea and abdominal pain. Early recognition and treatment of C. difficile infection is vital; always send stool for C. difficile in patients presenting with watery diarrhoea. Features of mild infection include <3 watery stools/ day (type 5-7 on Bristol stool chart) and no elevation of white cell count. Features of moderate infection include mild elevation of white cell count to <15 ×109/L and three to five watery stools/ day. Treatment for both is with oral metronidazole 400-500 mg tds for 10-14 days. Features of severe infection include: white cell count >15 ×109/L, or an acute rising serum creatinine (that is, >50% increase above baseline) temperature of >38.5°C, or evidence of severe colitis (abdominal or radiological signs). The number of stools may be a less reliable indicator of severity. Treatment is with oral vancomycin 125 mg qds for 10-14 days. Fidaxomicin should be considered for patients with severe CDI who are considered at high risk for recurrence; these include elderly patients with multiple comorbidities who are receiving concomitant antibiotics. In severe CDI cases not responding to oral vancomycin 125 mg qds, oral fidaxomicin 200 mg bd is an alternative, or high-dosage oral vancomycin to 500 mg qds, if necessary administered via a nasogastric tube, +/- IV metronidazole 500 mg tds is recommended. The addition of oral rifampicin (300 mg bd) or IV immunoglobulin (400 mg/kg) may also be considered. Features of life threatening infection include: hypotension partial or complete ileus or toxic megacolon, or CT evidence of severe disease. Treatment: oral vancomycin up to 500 mg qds for 10-14 days via nasogastric tube or rectal installation plus IV metronidazole 500 mg tds. Reference: Public Health England. Updated guidance on the management and treatment of Clostridium difficile infection.

Question 12 of 22 A 67-year-old woman is admitted to the Emergency Department with renal stones. She has a history of mild hypertension treated with ramipril, but nil else of note apart from a left Colles' fracture two years earlier. On examination her BP is 155/85 mmHg, pulse is 85 bpm and regular. She is in pain, specifically over her right renal angle. Abdominal x ray reveals multiple renal stones. Bloods reveal: Calcium 2.95 mmol/L (2.2-2.61) Creatinine 125 μmol/L (50-90) Phosphate 0.6 mmol/L (0.8-1.5) Which of the following is the most appropriate intervention? (Please select 1 option) Alendronate Bendroflumethiazide Cinacalcet Furosemide Parathyroidectomy

Parathyroidectomy Correct The blood picture here is consistent with primary hyperparathyroidism, and the patient has two indications to progress to surgery. The osteoporotic fracture and presentation with renal stones suggests parathyroidectomy is the most appropriate option. In four-gland hyperplasia, 3.5 glands are removed. Technetium imaging has significantly improved gland localisation prior to surgery and subsequent success in treating hypercalcaemia. Bisphosphonates have a role in the treatment of acute hypercalcaemia, and in the treatment of Paget's, and may be used to manage osteoporosis post-surgery, but not at this stage. Bendroflumethiazide reduces urinary calcium excretion and may have a role in reducing the risk of renal stones, but not in hyperparathyroidism. Cinacalcet is used in the treatment of tertiary hyperparathyroidism where surgery is not appropriate. Sevelamer is a phosphate binder used in the treatment of hyperphosphataemia related to chronic renal failure. Reference: Fraser WD. Hyperparathyroidism. Lancet. 2009;374:145-58.

Question 9 of 22 You are asked by the ST4 to review a 35-year-old alcoholic man who was admitted to the acute medical ward 36 hours earlier with symptoms of withdrawal. He is known to drink a bottle of Scotch whisky and four pints of strong lager per day. On examination he is drowsy and unwell. His BP is 90/60 mmHg, pulse is 95 bpm and regular. There are bilateral inspiratory crackles on auscultation of his chest consistent with heart failure and a global 4/5 power weakness. His BMI is 20.5. In total he has received approximately 1 litre of carbohydrate-rich supplement over the past 12 hours, Pabrinex A and B, and chlordiazepoxide. What intervention is likely to be most effective? (Please select 1 option) IV furosemide IV glucose Phosphate supplementation Stop chlordiazepoxide Thiamine supplementation

Phosphate supplementation This is the correct answer The symptoms seen here are consistent with re-feeding syndrome, where carbohydrate supplementation leads to reactive hyperinsulinaemia and a consequent fall in serum phosphate levels. This can precipitate drowsiness, confusion and even seizures. It also leads to global muscle weakness and can precipitate cardiac failure. Falls in potassium and magnesium are also seen. In the acute situation parenteral phosphate may be required, but this carries significant risk of arrhythmia and requires cardiac monitoring. Whilst IV furosemide will relieve fluid overload it is likely further to exacerbate hypotension and electrolyte abnormalities. He has already been given a carbohydrate-rich food supplement, IV glucose therefore is unlikely to have additional value. Thiamine supplementation is useful in the situation of heart failure due to wet beriberi, but this patient has already been treated with Pabrinex. Stopping chlordiazepoxide runs the risk of rapidly worsening symptoms of alcohol withdrawal.

Question 14 of 25 The following is a CT scan of a 77-year-old man presenting with pleuritic chest pain and shortness of breath: What was his likely occupation? (Please select 1 option) Airline pilot Cotton picker Factory worker Farmer Plumber

Plumber Correct It is important to be able to identify benign pleural calcification on a CT scan and be aware that previous exposure to asbestos may be a cause. In this image there is extensive calcification on the left pleura. Pleural plaques such as these can be the result of asbestos exposure. Other common causes of lung calcification include: True calcification: calcified pleural plaques from asbestos exposure: typically has sparing of costophrenic angles haemothorax infection involving the pleura - for example, pyothorax/empyema tuberculous pleuritis, and extra skeletal osteosarcoma of pleura (rare). Mimics of calcification: previous talc pleurodesis. Reference: Choi JA, Hong KT, Oh YW, et al. CT manifestations of late sequelae in patients with tuberculous pleuritis. AJR Am J Roentgenol. 2001;176:441-5. Leung AN, Müller NL, Miller RR. CT in differential diagnosis of diffuse pleural disease. AJR Am J Roentgenol. 1990;154:487-92. Murray JG, Patz EF Jr, Erasmus JJ, Gilkeson RC. CT appearance of the pleural space after talc pleurodesis. AJR Am J Roentgenol. 1997;169:89-91. Sabloff B, Munden RF, Melhem AI, et al. Radiologic-pathologic conferences of the University of Texas M. D. Anderson Cancer Center: Extraskeletal osteosarcoma of the pleura. AJR Am J Roentgenol. 2003;180:972.

Question 14 of 22 A 52-year-old woman who is known to have sarcoidosis is referred to the Emergency department for review with a calcium of 2.9 mmol/L, (2.2-2.61). She has suffered increasing thirst and polyuria over the past two to three weeks and has recently returned from a spring holiday to the Spanish coast. She is not currently taking corticosteroids. What intervention is likely to be most effective? (Please select 1 option) Calcitonin Cinacalcet Furosemide Pamidronate Prednisolone

Prednisolone Correct Hypercalcaemia in sarcoidosis is a result of increased production of vitamin D3 as a result of granulomatous inflammation. It is also now recognised that holiday hypercalcaemia may occur in sarcoidosis due to excess sun exposure and high consumption of phosphate- containing drinks, (that is, coca cola). A short course of prednisolone is enough to lead to a reduction in serum calcium if symptoms are severe, although for many patients keeping out of the sun and avoiding phosphate-containing drinks may be adequate intervention. Cinacalcet is used for tertiary hyperparathyroidism in patients who are unable to undergo parathyroid surgery. Calcitonin and pamidronate are both treatments for acute hypercalcaemia, but given a calcium of 2.9 mmol/L is not life-threateningly high, and prednisolone is likely to be effective, they are not the options of choice here. Furosemide is calciuric, but will of course worsen symptoms of polyuria.

Question 6 of 19 A 61-year-old lady presents in what you suspect is scleroderma renal crisis (SRC). She has a known history of scleroderma for which she has recently been started on prednisolone and is now presenting hypertensive at 220/100 mmHg, in acute kidney injury creatinine 330 mmol (baseline normal), and pulmonary oedema. Which of the following medications would you stop urgently? (Please select 1 option) Captopril Ciprofloxacin Paracetamol Prednisolone Simvastatin

Prednisolone Correct Scleroderma renal crisis (SRC) occurs in a small percentage of patients with systemic sclerosis. A typical presentation is that of an abrupt onset of: hypertension acute renal failure headaches fevers malaise hypertensive retinopathy encephalopathy, and pulmonary oedema. Mortality rates are high. SRC is more common in patients with a rapidly progressive or cutaneous form of scleroderma, though recently started high doses of corticosteroid therapy are a well recognised trigger for development of the condition.

Question 13 of 22 Core Questions You are called by a country GP who has received blood results on a 64-year-old woman who had a routine creatinine and HbA1c check before her annual review in one week's time. The results show a sodium of 138 mmol/L (135-145), potassium of 7.2 mmol/L (3.5-5.5), and are flagged as requiring urgent action. Creatinine is stable at 205 μmol/L vs. 195 μmol/L six months earlier. Medication includes insulin for type 1 diabetes, ramipril and atorvastatin What is the most likely diagnosis? (Please select 1 option) Addison's disease Pseudohyperkalaemia Renal artery stenosis Renal tubular acidosis type II Renal tubular acidosis type IV

Pseudohyperkalaemia This is the correct answer This is a relatively common situation related to difficulty taking blood leading to haemolysis, or blood having been left to stand for a long time before processing. You cannot be 100% certain this is the case however, so the patient should still be called in to the Emergency department to have a repeat potassium. The sodium is normal which makes a diagnosis of Addison's very unlikely. If this were renal artery stenosis, a deterioration in creatinine would be expected, as against the stable position seen here. Renal tubular acidosis type IV is associated with a deficiency of aldosterone action due to renal damage, a low sodium would be expected which is not seen here. Type II renal tubular acidosis is due to a defect of bicarbonate reabsorption and leads to acidosis coupled with disordered calcium and vitamin D metabolism, (osteomalacia).

Question 2 of 25 An 89-year-old lady has been brought in "off her legs" from a residential home. She can usually mobilise using a Zimmer frame, however for the past three days she has not been able to get about. She has also been complaining of back pain and that she "can't feel her feet". On arrival she is in urinary retention with a residual volume of 1.7 L. She is unable to tell you when she last opened her bowels. Her past medical history includes hearing impairment, right breast cancer (mastectomy, chemo and radiotherapy treatment), hypertension, hyper- cholesterolaemia, constipation, glaucoma, osteoarthritis, chronic kidney disease stage 3, ischaemic heart disease, previous TIAs, type 2 diabetes (diet controlled). On examination she is alert and oriented, GCS 15, complaining of back pain. Power is 2/5 in both lower limbs, there is increased tone, and a sensory level at T12. The plantars are both up going. Which of the following treatments are definitive management for this patient? (Please select 1 option) Bisphosphonate intravenously High dose dexamethasone intravenously High dose dexamethasone orally MRI brain Radiotherapy

Radiotherapy Correct This questions asks what treatment is definitive management for this patient. In reality the patient would be commenced on high dose dexamethasone orally (with proton pump inhibitor cover and bone protection) and an MRI Spine conducted to locate the offending lesion. Radiotherapy however is needed as soon as possible to try and shrink a spinal tumour and hence treat the compression. However, after three days, as in this patient's case, the chances of nerve regeneration are slim. High dose dexamethasone may help with surrounding oedema but is not definitive treatment. One should have a high level of suspicion of cord compression if a patient has a past medical history of cancer.

Question 22 of 29 A 31-year-old gentleman presents with an erythematous right elbow and fever. There is a large patch of erythema surrounding the entire elbow, and extending both up and down the arm. There is mild swelling of the elbow as well, with associated restricted range of movement. Initial investigations show a white cell count of 18 ×109/L and CRP of 250 mg/L. Elbow x ray shows no evidence of osteomyelitis. He is started on intravenous benzylpenicillin. Twenty-four hours later he develops increasing erythema throughout the right arm, as well as the left arm and throughout the trunk and back. The rash is macular papular, and there is an associated fever. Blood cultures from admission reveal a Group A Streptococcus. What is the most appropriate course of action? (Please select 1 option) Reculture and continue with benzylpenicillin Reculture and stop all antibiotics Reculture, continue with benzylpenicillin and add clindamycin Reculture, stop benzylpenicillin and start clindamycin Reculture, stop benzylpenicillin and start flucloxacillin

Reculture, continue with benzylpenicillin and add clindamycin This is the correct answer Group A Streptococcus septicaemia requires multidisciplinary management, including orthopaedic and plastic surgeons (in case of necrotising fasciitis) as well as infectious disease physicians. Treatment involves antibiotics, management of any arising organ dysfunction and surgical debridement if necessary. The initial choice of antibiotic is normally benzylpenicillin and clindamycin. The reason for having both benzylpenicillin and clindamycin is to cover for any resistance. Reference: UpToDate. Group A streptococcal (Streptococcus pyogenes) bacteremia in adults.

Question 18 of 22 A 39-year-old man is referred for review to the ambulatory care team. His GP is worried that his hypertension is increasingly difficult to control. Current medication includes amlodipine, indapamide and ramipril, yet his most recent BP is measured at 165/90 mmHg. He has chest tightness when walking up a steep hill near his home and anxiety, but no other symptoms of note. Examination in the admissions ward clinic room confirms the hypertension but is otherwise unremarkable, his BMI is 23. Investigations reveal: Hb 132 g/L (140-174) WCC 6.2 ×109/L (4-10) PLT 203 ×109/L (130-400) Na 141 mmol/L (135-145) K 3.2 mmol/L (3.5-5.5) Bicarbonate 32 mmol/L (24-30) Cr 112 μmol/L (70-120) Which of the following is the most appropriate next investigation? (Please select 1 option) 24 hour urinary catecholamines 24 hour urinary free cortisol Renal angiogram Renin:aldosterone ratio USS abdomen

Renin:aldosterone ratio This is the correct answer Difficult to control hypertension, including persistent hypokalaemia, despite the use of multiple agents including ACE inhibition, raises the possibility of hyporeninaemic hyperaldosteronism. In total it is thought to account for approximately 5-10% of all hypertensive patients, with a higher prevalence amongst those who attend clinics for resistant hypertension. The fact that this patient is of normal weight, has a normal creatinine and no significant PMH reduces the possibility of an alternative cause. Renin:aldosterone ratio is the initial investigation of choice as it is non-invasive. Traditionally it is recommended this is conducted off therapy, although given there is significant hypokalaemia and hypertension in this case, it is likely to be abnormal even on treatment. If positive it should be followed by abdominal imaging. The lack of obesity or impaired glucose tolerance reduces the likelihood of Cushing's significantly, as such urinary free cortisol is not useful here. Phaeochromocytomas often lead to episodic release of catecholamines, leading to paroxysms of headache and tachycardias, although if renin:aldosterone ratio proves unremarkable, it would be considered an alternative diagnosis here. Renal angiography is not a next investigation of choice here. In the case of renovascular disease, the renin:aldosterone ratio will prove useful as it would be expected to demonstrate hypereninaemic hyperaldosteronism. USS abdomen should be avoided before biochemical investigation because of the risk of detecting an adrenal incidentaloma.

Question 29 of 29 A 64-year-old man presents with unsteadiness, cough and fever. Initial investigations reveal a right lower lobe pneumonia, a CRP of 300 and white cell count of 15 ×109/L. His sodium is low at 124 mmol/L. His initial Mycoplasma, Chlamydia and Legionella serology were negative. He was treated with seven days of co-amoxiclav and clarithromycin as per local guidance for community acquired pneumonia and discharged. However, after completing treatment 11 weeks later, he re-presents with a cough, pyrexia and raised inflammatory markers. Urine dip is negative. Which of the following is the most useful investigation in order to determine the duration and choice of further antibiotics in this gentleman? (Please select 1 option) Repeat CRP Repeat Legionella serology Repeat white cell count Sputum for Legionella culture Urine Legionella screen

Repeat Legionella serology This is the correct answer Whenever atypical infection is suspected, always repeat serology in order to check for convalescence. Pneumonia is the main clinical manifestation of Legionella, and Legionella is a common cause of community acquired pneumonia. In the initial diagnosis, urine PCR or sputum culture are the preferred methods of diagnosis. Serology is not particularly useful in the acute phase, as confirmation of infection requires demonstration of convalescence.

Question 15 of 29 An 83-year-old lady presented to the Emergency department with abdominal pain and fever. She had recently returned from India and on closer questioning, had a three week history of abdominal pain, constipation and fever. There were no other unwell contacts, and she had not been to any other countries apart from India. There were no other symptoms on systems review. She was admitted under the medical team who managed her with IV fluids. On the fourth day of her admission she had an acute deterioration. She was complaining of severe abdominal pain and on examination had a tense abdomen. Her HR was 50 bpm, BP 80/70 mmHg, temperature of 38°C and her urine output dropped to 5 ml/hr. A venous blood gas showed: pH 7.12 BE −12 HCO3 10 Lactate 6 A CT abdomen at the time did not show any abnormality. Her urine output did not pick up and she was transferred to the ITU for filtration and noradrenaline. Blood cultures showed Gram negative rods, and she was started on ciprofloxacin. What is the most likely causative organism? (Please select 1 option) E. coli Enterobacter cloacae Proteus mirabilis Salmonella enteritidis Salmonella typhi

Salmonella typhi Correct This lady has typhoid/enteric fever, caused by Salmonella typhi or paratyphi. The most common symptoms are abdominal pain and fever in a patient returning from an endemic area such as south east Asia or southern Africa. One of the classic features of this infection is the relative bradycardia associated with the fever. Constipation tends to be more common than diarrhoea, particularly in adults. The incubation period is 5-21 days. In the absence of the relative bradycardia with the fever, and with diarrhoea, E. coli or Salmonella enteritidis would be a possibility for the cause of this lady's symptoms. Proteus mirabilis is normally associated with urinary tract infections.

Question 24 of 25 A 73-year-old Indian lady presents to you with fever, headache, confusion and left iliac fossa pain. Her past medical history includes osteoarthritis for which she takes glucosamine. She returned from a holiday in India four days ago. She travels to India every year and stays there for four months over the winter period. On examination she is pyrexial at 38.9°C, BP 110/60 mmHg, HR 53 bpm. She has been a little constipated recently but on the ward she has passed some loose watery stools. There is a macular non-pruritic rash over her trunk and limbs. CXR is NAD, urine is NAD and ECG shows a sinus bradycardia of 48 bpm. You send blood tests and blood cultures, whilst starting supportive management. What is the most likely organism? (Please select 1 option) Clostridium difficile Escherichia coli Giardia Rotavirus Salmonella typhi

Salmonella typhi This is the correct answer Salmonella typhi presents with high fever, rose spots, relative bradycardia, relative leucopenia and constipation or diarrhoea. Elderly patients often present in a non-specific manner and may easily become confused. This is a case of fever in a returning traveller. In the modern day society many 'elderly care' patients travel frequently and present with symptoms of fever and confusion. Remember to take a travel history and keep an open mind

Question 6 of 25 A 68-year-old man is admitted to you with a chest infection and a CURB score of 1. You are ready to discharge him when his son asks to speak with you. His son is concerned that his father is not as mentally sharp as he used to be since his retirement. It has been a rapid cognitive decline; his speech and motor skills seem slower, he is not disoriented but he does seem to have a short term memory loss. He no longer engages with his grandchildren with the energy and enthusiasm he used to have. From the above information all of the signs seem to indicate depression. Which of these signs is more consistent for dementia? (Please select 1 option) Less engagement with relatives Oriented to time and person Rapid cognitive decline Short term memory loss Slower speech and motor skills

Short term memory loss This is the correct answer Depression remains under-diagnosed in the elderly but there is growing recognition of the condition. There are simple tools available to aid diagnosis, such as the five point geriatric depression scale and the two question screener. Symptoms of depression tend to be quick in onset whereas cognitive impairment is a slow decline. Patients with depression may find difficulty in concentrating or engaging in fun activities, but should not suffer actual memory loss. If they do they are likely to be aware that their cognitive functions are suffering. Dementia patients often display memory loss and will lack insight. SSRIs are considered first line anti-depressive agents in the elderly although they can take between six to eight weeks to take effect. They have a better side effect profile, particularly if taken in overdose, compared with tricyclics. Reference: Helpguide. Depression in Older Adults and the Elderly.

Question 16 of 29 A 30-year-old gentleman from the Philippines presented with a three day history of headache, fever, mild photophobia, general abdominal pain and general malaise. There were no other localising symptoms on systems review. He had travelled to Croatia recently, but only to urban areas and was well during this time. His only past medical history was a diagnosis of ulcerative colitis, for which he was recently started on sulfasalazine. On examination, there were no localising findings. Initial investigations reveal nothing abnormal in full blood count, U&Es, or CT head. Lumbar puncture parameters are all within normal ranges. Liver function tests revealed: ALT 300 IU/L ALP 80 U/L Bilirubin 30 µmol/L CRP 4 mg/mol Brucella, Bartonella, Legionella and Mycoplasma serology all negative. Atypical pneumonia screen negative. What is the most likely diagnosis? (Please select 1 option) Brucellosis Legionnaires' disease Side effects of sulfasalazine Viral hepatitis Viral meningitis

Side effects of sulfasalazine This is the correct answer Always take a thorough medication history (including duration and compliance) and always consider medications as causes of symptoms and signs that do not always fit together. Additionally, always remember to take blood for convalescent serology for seroconversion. The most common side effects of sulfasalazine are nausea and gastric complaints. Other rarer side effects include fever, abdominal pain, aseptic meningitis and hepatitis. Reference: BNF: Sulfasalazine

Question 23 of 25 A 74-year-old lady is admitted acutely with a fall and confusion. On examination you note an abnormal gait with a tremor. She complains of feeling unsteady on her feet and it appears that she has been suffering with hallucinations. Her family mention symptoms consistent with amnesia and a steady decline in the patient's ability to care for herself. A CT of her head confirms age related atrophy and but shows no active lesions. You suspect a diagnosis of dementia with Lewy bodies. Which of the following would you request in order to confirm your suspicion? (Please select 1 option) Amyloid PET scan Apolipoprotein E4 gene (APOE4) Beta amyloid and tau protein levels MRI Head Single-photon emission computed tomography (SPECT)

Single-photon emission computed tomography (SPECT) Correct The patient is demonstrating general signs of dementia. Specific imaging may allow a more specific diagnosis of microvascular dementia, Alzheimer's or Lewy body dementia to be made. Amyloid PET scans use a tracer to detect a build up of fibrillar forms of cerebral amyloid which generally increases with age. APOE4 is a blood test used to identify candidates at increased risk of developing Alzheimer's disease but is not a diagnostic test. Low beta amyloid and elevated tau protein levels may be found in a diagnosis of Alzheimer's disease. Dopamine transporter imaging using single-photon emission computed tomography (SPECT) is used to support a diagnosis of DLB. Reference: Thies W, Bleiler L; Alzheimer's Association. 2013 Alzheimer's disease facts and figures. Alzheimers Dement. 2013;9:208-45. Walker RW, Walker Z. Dopamine transporter single photon emission computerized tomography in the diagnosis of dementia with Lewy bodies. Mov Disord. 2009;24:S754-9.

Question 21 of 22 Core Questions A 63-year-old lady presents to you having sustained a fracture of her femur after a fall. She is being assessed medically prior to going to theatre. She has a past medical history of type 2 diabetes, for which she is on metformin, and osteoporosis, for which she has been taking calcium and vitamin D supplements. Her HbA1c is 62 mmol/mol (7.8%). The patient informs you that she has stopped taking her gliclazide as it makes her prone to hypoglycaemic episodes. Which of the following therapies would you consider adding as a second line antihyperglycaemic agent? (Please select 1 option) Exenatide Levemir Liraglutide Rosiglitazone Sitagliptin

Sitagliptin Correct Do not commence thiazolidinediones in patients who have heart failure or are at higher risk of fracture. NICE guidance is very helpful in this area. Patients presenting with an HbA1c >48 mmol/mol (6.5%) require second line therapy to metformin. Sulphonylureas are a good second choice, except in those who have an allergy to this group of drugs or those suffering with hypoglycaemic episodes. Thiazolidinediones remain on the guidelines but should be prescribed with caution in the elderly due to multiple side effects. DPP4 inhibitors are a newer and generally safer class of drugs which can be added at first or second intensification. There is less chance of hypoglycaemia and less weight gain. Exenatide is a GLP1 mimetic, and is generally advocated as a third line agent to add on if HbA1c >58 mmol/mol (7.5%) and the patient shows a good metabolic response to the drug. Liraglutide is also a GLP1 mimetic, it is injected subcutaneously and should not be considered at first intensification. Reference: NICE. Type 2 diabetes in adults: management (NG28).

Question 15 of 22 Core Questions A 58-year-old man is admitted to the Emergency department with upper abdominal pain. He has a history of type 2 diabetes for which he takes a combination of metformin and sitagliptin, and mixed hyperlipidaemia treated with high dose atorvastatin. Examination reveals a BP of 135/75 mmHg, pulse is 90 bpm and regular. He is tender in the epigastrium. Lipase is elevated at 360 U/L (<160) and amylase is elevated at 490 U/L (24-30). Which of the following of his medications should be discontinued over the longer term? (Please select 1 option) Atorvastatin Bisoprolol Metformin Ramipril Sitagliptin

Sitagliptin Correct Sitagliptin has been noted as part of pharmacovigilance reporting to be associated with pancreatitis1. If pancreatitis is suspected, the sitagliptin should be discontinued. Many people recognise that GLP-1 agonists area associated with pancreatitis, but forget there have also been case reports of pancreatitis with DPPIV inhibitor use. The potential cause for pancreatitis is unclear, although it is known that GLP-1 inhibits secretion of a number of other gut peptides. Stopping the atorvastatin is ill-advised, as hypertriglyceridaemia is known itself to be associated with increased risk of pancreatitis. The other agents listed are not associated with pancreatitis, although temporary cessation of metformin is advised if there is a significant intercurrent illness which might increase the risk of tissue hypoxia. Reference: Electronic Medicines Compendium (eMC). JANUVIA 25 mg, 50 mg, 100 mg film-coated tablets: Undesirable effects.

Question 21 of 25 Core Questions A 39-year-old Asian lady with a history of epilepsy and psoriasis is admitted with weakness and abdominal pain. She is married but has no children. She is a smoker and is very thin. She moved to the UK seven years ago and is originally from the Philippines. Given her frail stature you are concerned she may have osteoporosis. Which of the following are risk factors for osteoporosis in pre-menopausal women? (Please select 1 option) Anticonvulsant therapy Asian ethnicity Nulliparous Psoriasis Smoking

Smoking Correct Women over the age of 65 or post-menopausal should be offered screening for osteoporosis. Women below the age of 65 years may also have risk factors for osteoporosis. Risk factors for low bone mineral density (BMD) include: Advanced age Previous fracture Long term glucocorticoid therapy Low body weight (less than 58 kg) Family history of hip fracture Cigarette smoking Excess alcohol intake, and Rheumatoid arthritis. Reference: Rosen CJ. Clinical practice. Postmenopausal osteoporosis. N Engl J Med. 2005;353:595-603.

Question 21 of 29 A 21-year-old Bangladeshi man presented with a four month history of intermittent thoracic back pain. He presented today as the pain was much sharper than previously. It was worse on mobility, and would radiate down his right hip. There were no associated symptoms of paraesthesia, loss of power, fever, and bladder or bowel dysfunction. On examination, he was afebrile, and had an antalgic gait. There were no abnormal neurological findings, and there were no focal areas of tenderness on examination of the back. Initial blood tests showed a CRP of 20 and a white cell count of 11 ×109/L. What is the most likely diagnosis? (Please select 1 option) Disc disease Musculoskeletal back pain Osteoarthritis Psoas abscess Spinal tuberculosis

Spinal tuberculosis Correct Inflammatory markers are rarely grossly elevated in tuberculosis. However, the slight elevation should point towards this diagnosis, as opposed to any of the others. Musculoskeletal tuberculosis accounts for 10% to 35% cases of extrapulmonary tuberculosis and for almost 2% of tuberculosis cases overall. Tuberculous spondylitis (Pott's disease) is the most common form of skeletal tuberculosis; it usually affects the lower thoracic and upper lumbar region. Infection begins with inflammation of the intervertebral joints and can spread to involve the adjacent vertebral body. Once two adjacent vertebrae are involved, infection can involve the adjoining intervertebral disc space, leading to vertebral collapse. Subsequent kyphosis can lead to cord compression and paraplegia. The most common symptom of tuberculous spondylitis is local pain, which increases in severity over weeks to months, sometimes in association with muscle spasm and rigidity. A characteristic erect posture and 'aldermanic' gait may be observed, in which the patient walks with short deliberate steps to avoid jarring of the spine. Constitutional symptoms such as fever and weight loss are relatively uncommon. The diagnosis of musculoskeletal TB is established by microscopy and culture of infected material. Tissue may be obtained by needle aspiration and/or biopsy, under CT or ultrasound guidance. The optimal duration of therapy for treatment of musculoskeletal tuberculosis is uncertain. For patients receiving treatment with first line agents in the absence of extensive or advanced disease, six months of therapy (rather than 9 or 12 months) are suggested. Reference: UpToDate. Skeletal tuberculosis.

Question 13 of 19 A 78-year-old man presents with a rash over his whole body which is very itchy. He has a history of chronic hyperuricaemia and has recently been started on allopurinol 200 mg once daily by his GP. You suspect this is an allergic reaction to the allopurinol. However, you also notice deposition of gouty tophi on the great toe and note a serum uric acid level of 600 mmol/L (120-450 mmol/L). Which of the following would you suggest for this man? (Please select 1 option) Prescribe long term antihistamine and continue allopurinol Reduce the dose of allopurinol Stop allopurinol and prescribe colchicine Stop allopurinol and prescribe febuxostat Stop allopurinol and prescribe prednisolone

Stop allopurinol and prescribe febuxostat This is the correct answer Febuxostat is relatively new on the market and is a non-purine selective inhibitor of xanthine oxidase. Febuxostat may be used to treat chronic hyperuricaemia in cases where allopurinol is contraindicated, for example in cases of drug allergy or mild-moderate renal impairment. Data are not available for severe renal impairment and manufacturers advise use with caution. Reference: NICE. Hyperuricaemia - febuxostat (TA164).

Question 10 of 19 A 56-year-old man presents with a dry cough but otherwise feels reasonably well. He has a history of psoriatic arthritis for which he is on methotrexate, non-steroidals and etanercept. On examination the patient is apyrexial. A mild kyphosis is apparent. Chest examination reveals normal vesicular breath sounds. You suspect the start of a lower respiratory tract infection. Blood tests are as follows: Hb 140 g/L WCC 10 ×109/L Plts 170 ×109/L CRP 58 mg Na 135 mmol/L K 4.3 mmol/L Urea 5 mmol/L Creat 89 µmol/L Which of the following would be the next best step? (Please select 1 option) Admit for intravenous antibiotics Change to adalimumab No treatment required Start oral antibiotics and discharge Stop etanercept and review

Stop etanercept and review Correct Anti-TNF therapy must be stopped at the first sign of infection. Etanercept will block naturally circulating tumour necrosis factor, which will mask symptoms of illness. In order accurately to determine the severity of illness, the patient will need to be re-assessed once therapy has been stopped. Tumour necrosis factor alpha is a cytokine activated by macrophages. One of its roles is to regulate the immune cells to induce fever and respond acutely to sepsis. It may well be that the patient is well and will not require admission, but observation over a short period of time off etanercept therapy will allow symptoms to develop. Antibiotics may then be prescribed as required. Change to adalimumab is not necessarily the answer in this case, it is also a biologic and will act to suppress the immune system. Treatment with etanercept can be restarted once infection has been treated. In the interim the psoriatic arthritis will require 'bridging treatment'. Specialist advice should be sought from the rheumatologist who commenced etanercept therapy.

Question 11 of 25 Core Questions An 89-year-old-man presents with a history of reduced appetite and non-specific abdominal pain. He also reports a foul smelling discharge from his back passage. On examination he is pyrexial but haemodynamically stable. The abdomen is soft, non-tender and the digital examination reveals a grey discharge which you send for culture. A urine dip is negative and his blood tests are as follows: Hb 150 g/L WCC 16 ×109/L Plts 500 ×109/L CRP 380 mg/L Na 134 mmol/L K 4.6 mmol/L Creatinine 171 μmol/L (baseline 74 μmol/L) Urea 15 mmol/L You decide to conduct a CT scan of the abdomen, shown below: How would you manage this patient? (Please select 1 option) Admit for intravenous antibiotics Discharge home with elderly care follow up Discharge home with gastroenterology follow up High fibre diet Surgical referral for an evaluation under anaesthesia

Surgical referral for an evaluation under anaesthesia Correct Simple diverticulitis is often admitted under the medical team as patients can be managed conservatively with intravenous antibiotics with Gram negative coverage. This patient presented with complications from his diverticulum and was diagnosed with fistula formation requiring surgical intervention. Possible complications of diverticulitis can include: peritonitis failed percutaneous drainage of an abscess enterocutaneous fistula formation, and bowel obstruction.

Question 9 of 19 You are asked by the orthopaedic surgeons to see a 73-year-old man who was brought to the Emergency department after a fall onto his left hip at the local supermarket. He is normally well, his only complaints being hypertension and mild gastro-oesophageal reflux disease, although on further questioning he tells you his left hip and pelvis have been painful for some months. On examination flexion and external rotation of the left hip are significantly limited due to pain. Bloods are unremarkable apart from the alkaline phosphatase which is markedly elevated at 441 U/L. Calcium and phosphate are normal. An x ray reveals extensive bony changes around the left hip and pelvis with osteolytic changes and new bone formation leading to significant joint deformity. Which of the following is the most appropriate treatment? (Please select 1 option) Calcitonin Cinacalcet Risedronate Teriparatide Vitamin D

The joint changes and blood picture with an isolated rise in alkaline phosphatase are consistent with a diagnosis of Paget's disease. The objective of bisphosphonate therapy is symptom control; studies suggest there is little benefit in therapy for patients with elevated alkaline phosphatase levels in the absence of significant bony pain. Calcitonin is a secondary therapy used for the treatment of Paget's in patients who cannot tolerate bisphosphonates. Cinacalcet is a calcimimetic used in the treatment of patients with tertiary hyperparathyroidism who are unsuitable for parathyroid surgery. Teriparatide induces osteoblast activity and is used as a treatment for severe osteoporosis. Vitamin D is used for the treatment of osteomalacia, although with a normal calcium and taking into account the radiology, there is no evidence this patient has osteomalacia.

Question 13 of 29 Core Questions A 28-year-old man who has sex with other men (MSM) presents acutely unwell with symptoms of cough, haemoptysis and fever. He is found to have florid consolidation on his chest radiograph. As part of his initial investigations, you would like to send off an HIV test. What must he be told prior to testing as part of pre-test counselling? (Please select 1 option) He cannot be prosecuted if he has unprotected sexual contact with others without disclosing his HIV status, despite being informed of the importance of disclosure to sexual partners He will definitely require lifelong treatment If positive, he need not tell his insurance company The result can be passed to a relative or friend, without the consent of the patient, or without informing the patient that this is going to be done There is a potential need for a repeat test three months after a potential exposure

There is a potential need for a repeat test three months after a potential exposure This is the correct answer HIV is now a treatable medical condition and the majority of those living with the virus remain fit and well on treatment. Despite this, a significant number of people in the United Kingdom are unaware of their HIV infection and remain at risk to their own health and of passing their virus unwittingly on to others. Patients should therefore be offered and encouraged to accept HIV testing in a wider range of settings than is currently the case. Patients with specific indicator conditions should be routinely recommended to have an HIV test. If the test is positive, as with any other medical condition, the patient's insurance provider must be informed. UK law states that prosecutions can take place under general assault laws (reckless/ intentional transmission or grievous bodily harm) of those who may be HIV positive and are not informing their partners. Factors such as viral load and use of barrier protection are taken into account. Reference: British Association of Sexual Health and HIV (BHIVA). UK National Guidelines for HIV Testing 2008. The Joint United Nations Programme on HIV and AIDS (UNAIDS). Criminalisation of HIV Non-Disclosure, Exposure and Transmission: Background and Current Landscape.

Question 11 of 22 A 78-year-old woman presents with confusion for the third time over the past nine months. According to her relatives she has become progressively more drowsy and muddled over the past two weeks. A past history of type 2 diabetes, hypertension, ischaemic heart disease and unstable bladder is noted. Medication includes ramipril, amlodipine, tolterodine, metformin and mixed insulin. Her BP is 148/82 mmHg, pulse is 80 bpm and regular. She is not in cardiac failure. Investigations reveal: Na 122 mmol/L (135-145) K 4.5 mmol/L (3.5-4.5) Cr 122 μmol/L (50-90) Glucose 12.4 mmol/L (<11.5) TSH 2.4 mU/L (0.5-4.5) What is the most likely cause of her measured low sodium? (Please select 1 option) Cerebrovascular disease Fluid overload Metformin Pseudohyponatraemia Tolterodine

Tolterodine Correct Tolterodine is used for the treatment of bladder instability and is a rare cause of syndrome of inappropriate antidiuretic hormone (SIADH). The mechanism by which this occurs is unclear; it may either increase ADH release or potentiate its effects. In this case, with recurrent SIADH in the absence of other factors which may cause it, a pharmaceutical cause should be considered. Whilst chronic small vessel cerebral ischaemia can result in hyponatraemia, we are not told of any evidence of neurological deficit which would fit with this as a cause. Equally, there is no evidence at all of fluid overload. Metformin is not known to cause SIADH, and a glucose of 12 mmol/L is not of the required level to result in pseudohyponatraemia. Reference: Ustun I, Davarcı M, Demirbaş O, et al. Recurrent hyponatremia due to tolterodine. Can Urol Assoc J. 2012;6:E129-30.

Question 8 of 22 A 38-year-old woman who works in a pharmacy is admitted to the Emergency department after an episode of collapse at work. It was a particularly hot day, she felt faint whilst making up prescriptions in the pharmacy and then passed out. This is a third admission in the past six months. She was treated for an eating disorder as a teenager, but there is no other past medical history of note. On examination her BP is 95/60 mmHg, pulse is 82 bpm and regular. There are no abnormal findings and her BMI is 21. Investigations reveal: Hb 132 g/L (123-157) WCC 7.2 ×109/L (4-10) PLT 190 ×109/L (130-400) Na 141 mmol/L (135-145) K 3.1 mmol/L (3.5-5.5) Bicarbonate 36 mmol/L (24-30) Cr 92 μmol/L (50-90) Which of the following is the most appropriate next investigation? (Please select 1 option) Genetic analysis Random cortisol Renin:aldosterone ratio Urine calcium Urine diuretic screen

Urine diuretic screen This is the correct answer There are clues in the history, clinical findings and laboratory work up that indicate the likely underlying diagnosis. Key points from the history indicate this patient has a previous history of laxative abuse and works in a pharmacy. This is coupled with hypokalaemic metabolic alkalosis. The obvious implication is therefore that she is abusing diuretics. The alternatives would be laxative abuse (but we are given no history of GI upset), Bartter's or Gitelman's syndrome. With respect to Bartter's syndrome, patients usually present with severe symptoms in childhood. Gitelman's can present later, with much milder symptoms, and would be an alternative diagnosis here. Transparency with respect to performing the drug screen is of course crucial to this patient's recovery. Urine calcium is useful in differentiating between Gitelman's and Bartter's (calcium excretion is low in Gitelman's, and is high in Bartter's). Genetic analysis is only useful in the event that Bartter's or Gitelman's is suspected as the underlying cause of the hypokalaemic metabolic alkalosis. In reality it would be considered after other potential causes have been excluded. The low-normal blood pressure and normal weight makes Cushing's very unlikely, hence cortisol does not have great utility in this case. Similarly, renin:aldosterone ratio is useful in the diagnosis of Conn's, although she is not hypertensive.

Question 18 of 25 You are about to discharge an 89-year-old man who was admitted with a non-infective exacerbation of COPD. Whilst in hospital as an inpatient, he has seen posters promoting the annual influenza vaccine for his age group. He is afraid of having the vaccination and asks for your advice. What would be the most appropriate advice for your patient? (Please select 1 option) Vaccination is not required Vaccination is required and it offers 100% protection against influenza Vaccination is required and will prevent respiratory syncytial virus (RSV) Vaccination is required but it may result in flu-like symptoms Vaccination will help to reduce onset of bacterial pneumonia

Vaccination will help to reduce onset of bacterial pneumonia This is the correct answer It is important that healthcare professionals are aware of the recommendations surrounding influenza that has been the cause of many epidemics and deaths globally. The World Health Organisation (WHO) and US Centers for Disease Control and Prevention (CDC) monitor the various changing strains of influenza and this results in a new vaccine annually. There are type-A and type-B strains, with the notorious H1N1 being a type-A strain. There are certain features associated with influenza that make it more virulent when compared to other viral infections. Firstly it tends to descend into the lower respiratory tract and destroy lung cells, which results in poor pulmonary function. Secondly, it tends to promote bacterial superinfection resulting in pneumonia, particularly in those who are immunocompromised (for example, elderly, pregnancy). The vaccination does not offer protection against other viral infections such as the respiratory syncytial virus, the most important cause of respiratory disease in the elderly. No vaccination can claim to protect 100% of recipients, but epidemiological studies have shown that mass vaccination results in a lower incidence of influenza overall. Reference: Medscape Multispecialty: Seasonal Influenza Update

Question 7 of 29 A 60-year-old gentleman presented with a few days' history of headache and fever. Clinically, there was no photophobia, neck stiffness, rash or any other signs of meningism. On examination, his heart, lungs and abdomen were all NAD. Full neurological examination revealed a subtle bilateral lateral rectus palsy. Blood tests showed: WCC 7 ×109/L CRP 3 mg/L Platelets 130 ×109/L U&Es normal LFTs normal A non-contrast CT head showed no obvious abnormality. He proceeded to have a lumbar puncture, which showed: Protein 0.34 mg/L/L Glucose 6 mmol/L Serum 11 mmol/L RCC 0 cells/mm3 WCC 0 cells/mm3 Cytology No atypical cells Opening pressure >40 mmHg Xanthochromia was negative What is the most likely diagnosis? (Please select 1 option) Encephalitis Meningitis (viral or bacterial) Space occupying lesion Subarachnoid haemorrhage Venous sinus thrombosis

Venous sinus thrombosis This is the correct answer This patient has multiple venous sinus thromboses, as was later picked up on his MRV scan. He has no symptoms of meningism, and there is nothing to suggest the acute confusion that would be suggestive of encephalitis. His neurological examination showed bilateral lateral rectus palsies; CN 6 palsies are often false localising signs secondary to raised intracranial pressure (ICP) and this gentleman had raised ICP from his multiple venous sinus thromboses. The symptoms of venous sinus thrombosis will vary depending on the location of the clot. There are a variety of causes/associated factors, including: sinusitis hypercoagulable states (including primary hypercoagulable states such as antiphospholipid syndrome, protein C/S deficiencies, as well as malignancy) trauma surgery drugs (for example, oral contraceptive pill, steroids, EPO, tamoxifen), and other conditions (inflammatory bowel disease, pregnancy, SLE, nephritic syndrome and hepatic cirrhosis). Symptoms include: headache nausea vomiting focal neurological deficit, and diplopia. Treatment mainly involves anticoagulation, but care should be taken in the initial treatment phase as there is a risk of haemorrhage. In meningitis one would expect some white cells in the CSF (mainly lymphocytes in viral, and polymorphs in bacterial). Subarachnoid haemorrhage is a possibility, however the headache is normally 'thunderclap' in nature and the xanthochromia is negative. Space occupying lesion (SOL) is also a possibility in the context of the raised opening pressure and false localising sign, but a SOL that large would show up on a plain CT scan.

Question 10 of 25 A 83-year-old man with a history of cerebral vascular accident (CVA) is repatriated from your local hyperacute stroke unit (HASU) for rehabilitation. The nurses inform you that he is demonstrating signs of oropharyngeal dysphagia. Which of the following would you request as an initial investigation? (Please select 1 option) Barium swallow Fiberoptic endoscopic evaluation of swallowing Manometry Upper gastrointestinal endoscopy Video fluoroscopy

Video fluoroscopy Correct Managing oropharyngeal dysphagia includes: taking a detailed history to determine the underlying cause (neurological/mechanical) choosing the correct investigation to visualise the problem, and providing rehabilitation as appropriate to improve ingestion of food and reduce episodes of aspiration. Each of the answer options is one of the five major tests which can be used to evaluate oropharyngeal dysphagia. However, the initial investigation for oropharyngeal dysphagia secondary to neurological injury is video fluoroscopy.

Question 17 of 25 You are asked to assess an 83-year-old lady who is being 'troublesome' on the elderly care ward. She has been spitting at and punching the nurses. She had initially been admitted for a urinary tract infection, which has now been treated and is currently awaiting a care package. The nurses are keen to restrain her physically using mittens and soft cot sides. Which of the following is true with regard to patient restraint? (Please select 1 option) Patient restraint is no longer legal You can restrain a patient if they are being aggressive and unmanageable You can restrain a patient if they are refusing medication/treatment You cannot restrain a patient if force is required to do so You cannot restrain a patient who has capacity

You cannot restrain a patient who has capacity Correct The Mental Capacity Act (MCA) allows the lawful (physical) restraint of a person lacking capacity although it should always be a last resort. Restraint has become a hot topic as it verges on 'physical abuse' if not applied in the correct circumstances. Many hospitals now insist on a senior nurse or consultant review to assess need for manual restraint. The definition of restraint includes the restriction of liberty of movement, whether or not the person resists or the use of threat or force to make someone do something when they are resisting. Criteria for applying restraint may vary in each local trust. It can include situations where: the person lacks capacity and restraint would be in their best interest it is reasonable to believe that restraint is necessary to avoid harm to the patient, and the degree of restraint is proportionate to the response of the patient.


Related study sets

BUAD 201 Chapter 2 Practice Quiz

View Set

Chapter 8 Alcohol and Tobacco Use and Abuse

View Set

Civil Liberties and Civil Rights Study Guide

View Set

Maternal-Newborn Chapter 18 The Newborn

View Set